Gases Ideales y Reales

Autor: Amado Crisógono Castro Chonta 2015-2 CONTENIDO LAS PROPIEDADES DE LOS GASES SE EXPLICAN MEJOR MEDIANTE LA TEORÍ

Views 151 Downloads 9 File size 3MB

Report DMCA / Copyright

DOWNLOAD FILE

Recommend stories

Citation preview

Autor: Amado Crisógono Castro Chonta

2015-2

CONTENIDO LAS PROPIEDADES DE LOS GASES SE EXPLICAN MEJOR MEDIANTE LA TEORÍA CINÉTICO MOLECULAR………………………………………………………………….…..…. ….3 I) GASES IDEALES………………………………………………………….............5 LEYES DE LOS GASES IDEALES ……………………………………………………………….…………..….6 LEY

DE

BOYLE………………….

…………………………………………………………………………..……..6 LEY DE GAY-LUSSAC Y CHARLES……………………………………………………………………..…….10 PRIMER CASO: LEY DE LAS ISOBÁRICAS…………………………………………..……..….…..10 SEGUNDO CASO: LEY DE LAS ISOMÉTRICAS………………………. …….………………………13 ECUACIÓN DE ESTADO DEL GAS IDEAL………………………………………………………….………..17 MEZCLAS DE GASES IDEALES…………………………………………………………………. …………….26 LEY DE LAS PRESIONES PARCIALES DE DALTON…………………………………….………..…26 LEY DE LOS VOLÚMENES PARCIALES DE AMAGAT……………………….…………….……...28

DETERMINACIÓN DE LAS MASAS MOLECULARES DE LAS SUSTANCIAS DE BAJA TEMPERATURA DE VAPORIZACIÓN…………………………………………………….………….…39 LEY DE GRAHAM DE LA DIFUSIÓN……………………..

……………………………….…….………42 II) GASES REALES ………………………………………………….………….…..46 LAS ECUACIONES DE ESTADO............................................................................................... 46 ECUACIONES DE ESTADO DE VDW……….…………………….

…………………………………..47 FACTORES DE COMPRESIBILIDAD…………………………. ……………………………………53

AMADO DE DIOS ([email protected])[Escriba texto] Página 1

Autor: Amado Crisógono Castro Chonta MEZCLAS

GASEOSAS

DE

LOS

GASES

2015-2 REALES ………………………..

……………..……………………..64 ECUACIÓN DE ESTADO…………………………………………………………………………………

….64 CONSTANTE PROMEDIO……………………………………………………………………,,….

……….65 FACTOR DE COMPRESIBILIDAD

MEDIO…………………………………………………..…….…..65 PROPIEDADES SEUDOCRÍTICAS……………………………………………………………...

………..66 BIBLIOGRAFIA ………………………………………………………………………………….………. ………..72 TABLA DE FACTORES DE CONVERSIÓN…………….. ………………………………………………….77 PROBLEMAS …………………………………………………………. …………………………………………….82 ALGUNAS ECUACIONES UTILIZADAS………………………………………..…………………………...71

DIAGRAMAS Y GRÁFICOS: Página Diagrama Nº1: Expansión de una masa de gas a temperatura constante…….………..3 Diagrama Nº 2: “Ley de las isotermas del gas ideal de Boyle………………..………..6 Diagrama Nº 3: Representación de las isotermas T1 < T2 < T3 de un Gas Ideal en los diagramas P vs V y T vs p……………………………………………..……….….….8 Diagrama Nº 3,1: El gráfico de la isoterma en el diagrama P vs. V, para este problema es: ………………………………………………………………….…………….……..9 Diagrama Nº 4: Ley de las isóbaras de un GI de Jacques Charles….…………….…...10 Diagrama Nº 5: Comportamiento del GI en el gráfico V vs. t…….…………….…….11 Diagrama Nº 6: Las isóbaras de Jacques Charles……………………………….……..12 Diagrama Nº 7: Las Isométricas de Joseph-Louis Gay-Lussac.....................................13 Diagrama Nº 8: Gas Ideal encerrado en un cilindro-émbolo, cuyo

AMADO DE DIOS ([email protected])[Escriba texto] Página 2

Autor: Amado Crisógono Castro Chonta

2015-2

volumen puede variar al mover el émbolo………………….……….17 Diagrama Nº 9: De las presiones parciales de Dalton…………………………….…..26 Diagrama Nº10: El agua se transforma en vapor a 100ºC y 1 atm de presión….……..39 Diagrama Nº11: Ley de efusión de Graham…………………………………………..42 Figura Nº1: Comprobación experimental de la relación Pr/Tr = Cte/Vr para un conjunto de gases………………………………………………………………………………...54 Figura Nº2: Factor de compresibilidad como función de la temperatura y presión……55 Figura Nº3: Comprensibilidad como una función de la temperatura reducida y presión reducida………………………………………………………………………………...55 Figura Nº3,6a: Gráfica del factor de compresibilidad generalizado……………………73 Figura Nº3,7: Gráfica del factor de compresibilidad generalizado para presiones bajas…………………………………………………………………………………….74 Figura Nº3,8: Gráfica del factor de compresibilidad generalizado para presiones término medio…………….…………………………………….…………………………….…75 Figura Nº3,9: Gráfica del factor de compresibilidad generalizado para presiones altas……………………………………………………………………………………..75 Figura Nº3,10: Gráfica del factor de compresibilidad generalizado por escalas especiales…………………………………………………………………………….…76

GASES IDEALES Y GASES REALES 2,0 LAS PROPIEDADES DE LOS GASES SE EXPLICAN BIEN MEDIANTE LA TEORÍA CINÉTICA MOLECULAR:

AMADO DE DIOS ([email protected])[Escriba texto] Página 3

Autor: Amado Crisógono Castro Chonta

2015-2

Para comprender bien las propiedades de los gases y su comportamiento cuando cambian las condiciones experimentales; como: ¿Por qué se expande un gas cuando se calienta a presión constante? ¿O por qué aumentea su presión cuando el gas se comprime a temperatura constante?; es necesario un modelo que nos ayude a visualizar. Este modelo se llama Teoría Cinética-molecular, que fue desarrollado a lo largo de 100 años y que culminó en 1857 cuando Rudolf Clausius (1822-1888) publicó en forma completa y satisfactoria esta teoría. La Teoría Cinética- molecular (que trata de las moléculas en movimiento) se resume en los enunciados siguientes: 1. Los gases están constituidos por partículas muy pequeñas llamadas moléculas o átomos. 2. Las partículas de un gas están en constante movimiento, incesante y caótico ocupando todo el volumen disponible (V). Estas partículas chocan entre si y con las paredes del recipiente que los contiene. 3. Se puede transferir energía entre las moléculas durante los choques, pero la energía cinética media de las moléculas (Ec) no cambian con el tiempo, siempre que la temperatura del gas permanece constante, es decir los choques son perfectamente elásticos.

AMADO DE DIOS ([email protected])[Escriba texto] Página 4

Autor: Amado Crisógono Castro Chonta

2015-2

4. La energía cinética media de las moléculas (Ec) es proporcional a la temperatura absoluta (T); y a cualquier temperatura dada, las moléculas de todos los gases tienen la misma energía cinética media: (Ec T). 5. La presión de un gas (P) es el resultado de estos choques contra las paredes del recipiente y tienen el mismo valor sobre toda las paredes. 6. Si se reduce el volumen (V), aumentará el número de choques por unidad de área, aumentando la presión del gas (P), entonces concluimos que el volumen y la presión son inversamente proporcionales

 

 V

1  p 

para una misma masa de gas (m) a

temperatura constante (T). 7. Bajo condiciones ordinarias de presión (P) y temperatura (T) el espacio ocupado propiamente por las moléculas (Vpp) dentro de un volumen gaseoso, es una porción muy pequeña del volumen total del gas. Por ejemplo una molg de un gas ideal a condiciones normales de presión y temperatura, CNPT, ocupa un volumen de 22,414 L = 22 414 mL= 22 414 cm 3; y una molg de agua líquida, H 2O(l) , ocupa un volumen de 18mL=18cm3, que se calcula a partir de la densidad del agua: m 

1g 1mo lg 1mo lg ( ) entonces 1molg agua= 18 mL=18cm3 3 cm 18 g 18cm 3

que al analizar concluimos que el volumen del líquido es despreciable al comparar con el volumen del gas, y también que el volumen molar y la densidad molar son inversamente proporcionales

v n (l ) 

1  18cm 3 / molg m

Esto significa que en el gas la distancia de separación entre las moléculas son muy grandes al comparar con sus diámetros moleculares, entonces las fuerzas de atracción molecular en el gas se puede considerar despreciable. Esta es una de las propiedades de los GASES IDEALES. 8. Las moléculas de un gas por estar en constante movimiento, poseen energía cinética, que depende de la temperatura. Un aumento en la temperatura aumenta la energía cinética de cada molécula y si se mantiene el volumen constante aumentará la presión de dicho gas, entonces a mayor temperatura mayor presión cuando el proceso es a volumen constante para una misma masa de gas.

AMADO DE DIOS ([email protected])[Escriba texto] Página 5

Autor: Amado Crisógono Castro Chonta

2015-2

9. Las moléculas poseen fuerzas de atracción que trata de mantenerlas juntas y fuerzas de repulsión que está en función de su energía cinética que trata de dispersarlas. Estos no tienen el mismo valor para diferentes estados físicos de la materia: en los sólidos la fuerza de atracción alcanza su máximo valor; en los líquidos las fuerzas de atracción y repulsión son aproximadamente iguales y en los gases la fuerza de cohesión alcanza su valor mínimo, por estar las moléculas dotadas del máximo movimiento (energía cinética). 10. Algunas sustancias que se encuentran como gases 1 atm y 25ºC: Elementos: H2, N2, O2, O3, F2, Cl2, He, Ne, Ar, Kr, Xe, etc. Compuestos: HF, HCl, HBr, HI, CO, CO2, NH3, NO, NO2, N2O, SO2, H2S, CH4, HCN (punto de ebullición es 26ºC), etc. 11. Los gases se comportan de dos maneras, como: A) Gases Ideales y B) Gases Reales

I)

GASES IDEALES

El objetivo es estudiar el comportamiento de los gases como gases ideales: leyes, postulados y propiedades, y estas se expresan por la ecuación de estado de los gases ideales. Los Gases Ideales presentan tres propiedades importantes: primero el volumen propio de sus moléculas son despreciables con respecto al gran volumen del recipiente que la ocupan, y segundo la fuerza de atracción de sus moléculas son despreciables por que la distancia de separación entre sus moléculas son muy grandes, y tercero sus moléculas presentan choques elásticos entre sí y con las paredes del recipiente. Una colisión elástica es aquella en la cual no hay cambio en la energía cinética total; en consecuencia, las moléculas no absorben energía en los movimientos internos y las paredes tampoco absorben energía del gas. Las moléculas de los gases reales se atraen y se repelen entre sí en una pared que se describirán más adelante, y las reacciones químicas son ejemplos de colisiones inelásticas. Una ecuación de estado relaciona la cantidad molar y el volumen de un gas con la temperatura y presión. La relación más simple y usada de éstas es la ecuación de estado de los gases ideales (PV=nRT), la cual, aunque es aproximada, resulta adecuada para

AMADO DE DIOS ([email protected])[Escriba texto] Página 6

Autor: Amado Crisógono Castro Chonta

2015-2

muchos cálculos de ingeniería donde participa gases a baja presión y temperatura moderadamente alta

2,1 LEYES DE LOS GASES IDEALES: 2, 1, 1 LEY DE BOYLE

Diagrama Nº 2: “Ley de las isotermas del gas ideal de Boyle. Llamada también “Ley de las Isotermas” por ser un proceso a TEMPERATURA CONSTANTE. El sistema termodinámico es el gas ideal que está encerrado dentro del cilindro-émbolo. En el gráfico observamos si a temperatura constante (T) se duplica el volumen (V) de una misma masa de gas ideal (m), entonces la presión del gas (P) disminuye a la mitad y viceversa. Este comportamiento del gas ideal expresa que el volumen y la presión son inversamente proporcionales para una misma masa de gas a temperatura constante:  1   P

V  KB 

( V

1 )T,m y su ecuación termodinámica es: P

entonces (V.P)T,m = KB = constante de Boyle…… (1)

vemos que el producto de la presión del gas por su volumen para una misma masa de gas ideal a temperatura constante da como resultado un valor constante, llamada Constante de Boyle (KB) en honor a su descubridor Robert Boyle. Para cambios

AMADO DE DIOS ([email protected])[Escriba texto] Página 7

Autor: Amado Crisógono Castro Chonta

2015-2

finitos de estado termodinámico de una misma masa de gas ideal a temperatura constante, si un gas pasa de un estado (1) a un estado (2) tenemos: Estado 1:

(V1P1)T,m = KB

y

Estado2 :

(V2P2)T,m = KB

Para un mismo proceso termodinámico el valor de KB es constante, entonces: Igualando obtenemos

(KB = V1P1 = V2P 2 ) T,m ……………..……. (2)

Para el estado n se tiene

(VnPn = KB)T,m …………………...………..… (3)

Igualando las ecuaciones (2) y (3) tenemos:

(V1P1 = V2P2 =… =VnPn= KB)T,m…..............................(4) Manteniendo la temperatura constante para una misma masa de un gas ideal, siempre el producto de la presión por su volumen respectivo es un valor constante, igual a la Constante de Boyle (KB). La ecuación (4) también se puede expresar como: (

V1 P  2 )T,m V2 P1

y

(

V2 Pn  ) Vn P2 T,m

y

(

V1 P  n )T,m ………(5) Vn P1

También la ecuación de Boyle se puede expresar en función de sus densidades () Sea 1 para el estado 1 y 2 para el estado 2 de una misma masa de gas ideal (m) a temperatura (T) constante: 

 masa m V  volumen

entonces

(m = 1 V1)T

es el estado 1

(m = 2 V2)T

es el estado 2

Como la masa del gas es la misma y la temperatura es constante, podemos igualar (m = 1V1   2V2 )T  (

Pero

1 V2  ) ……………. (6)  2 V1 T,m

( V2  P1  1  P1 V1 P2 2 P2

)T,m…………….(7)

En conclusión para una misma masa de gas a temperatura constante la densidad de un gas ideal es directamente proporcional a su presión, ecuación (7), e inversamente proporcional a su volumen, ecuación (6) Un gas es ideal cuando las fuerzas de cohesión son cero y cuando el volumen propio de las moléculas es despreciable, es decir cero, frente al volumen total del recipiente ocupado por el gas. Esta ley no se cumple para los gases reales, salvo a presiones bajas y temperaturas moderadamente altas.

AMADO DE DIOS ([email protected])[Escriba texto] Página 8

Autor: Amado Crisógono Castro Chonta

2015-2

Diagrama Nº 3: Representación de las isotermas T1 < T2 < T3 de un Gas Ideal en los diagramas P vs V y T vs P. P

T T3 T3 T2

KB3 T2

KB2

T1

T1

KB1 V (PV)T,m = KB

Isotermas de un gas ideal

P

Las isotermas de un gas ideal en un diagrama P vs V, son asíntotas a los eje de la presión (P) y del volumen (V). Y son rectas paralelas al eje de la presión en el diagrama T vs P. Para cada isoterma el valor de la constante de Boyle son diferentes. PROBLEMA Nº 1: Una masa de gas ideal tiene un volumen de 20 L y una presión de 750 mmHg. Si la temperatura permanece constante cuando su presión varía sucesivamente a 600, 300, 200 y 100 mmHg aplicando la ecuación de estado de la Ley de Boyle trazar su ISOTERMA. Solución del problema : 1. Calcule primero la constante de Boyle (KB): (PV = KB)T,m

V1 = 20 L

P1 = 750mmHg

m = constante

T1 = T2 = T3 = T4 = T5 = T = constante (20L) (750 mmHg) = KB entonces 15 000 L.mmHg = KB 2.

Calcule los volúmenes: V2, V3, V4, V5 para cada variación de Presión : ( V2 

KB )m,T P2

V2 

15 000 LmmHg  25 L 600 mmHg

V3 

15 000 LmmHg  50 L 300 mmHg

V4 

15 000 LmmHg  75 L 200 mmHg

AMADO DE DIOS ([email protected])[Escriba texto] Página 9

Autor: Amado Crisógono Castro Chonta V5 

2015-2 15 000 LmmHg  150 L 100 mmHg

Diagrama Nº 3,1: El gráfico de la isoterma en el diagrama P vs. V, para este problema es:

P (mmHg) 800

P(mmHg) 750

V(L) 20

750

600

25

300

50

600

200

75

500

100

150

400

RESPUESTA:

300

La curva es la isoterma de este problema.

200 100

V(L)

20 25

50

75

100

125

150

PROBLEMA Nº 2: Se desea averiguar la presión adicional en otro que se debe proveer al gas a la presión de una atmósfera contenida en un globo, para obligarle a llenar otro globo cuyo volumen es ½ del volumen del primero, con el objeto que al final quede todo el sistema a la presión de una atmósfera. Robert Boyle (Lismore, 25 de enero de 1627 - Londres, 30 de diciembre de 1691) físico y químico irlandes, principalmente conocido por su formulación de la Ley de Boyle-Mariotte. Es considerado uno de los padres de la química moderna.

http://www.juntadeandalucia.es/averroes/recursos_informaticos/andared02/leyes_g ases/ley_boyle.html. http://platea.pntic.mec.es/~cpalacio/boyle2.htm. http://www.youtube.com/watch?v=GLlivXIIYiY

AMADO DE DIOS ([email protected])[Escriba texto] Página 10

Autor: Amado Crisógono Castro Chonta

2015-2

2, 1, 2 LEYES DE GAY – LUSSAC Y CHARLES Primer caso: “LEY DE LAS ISÓBARAS” variación del volumen (V) de una misma masa (m) de gas ideal con la temperatura (T) a presión (P) constante. Segundo caso: “LEY DE LAS ISÓCORAS O ISOMÉTRICAS” variación de la presión (P) de una misma masa (m) de gas ideal con la temperatura (T) a volumen (V) constante. PRIMER CASO: “LEY DE LAS ISÓBARAS”

Diagrama Nº 4: Ley de las isóbaras de un GI de Jacques Charles. A presión constante el volumen de una misma masa de cualquier gas ideal aumenta en 1/273,15 = 0,003 66 =  de su volumen a 0ºC, por cada grado centígrado de elevación en la temperatura. Sea (V0) el volumen de una determinada masa de gas a 0ºC, y (Vt) su volumen a la temperatura (t) expresada en grado Celsius, estará dado por la ecuación: Vt = Vo + V

V = Vo t

pero

Combinando las dos ecuaciones tenemos:

Vt = Vo + Vo t ...................(8) 1

donde   273,15 Factorizando es



y reemplazando tenemos: Vt  Vo  1 

Vt = Vo ( 1 + t)



 273,15  t    273,15 

Vt  Vo 

Pero

273,15 + t (ºC) =T (K)

t   273,15 

…………………………….(8a)

es temperatura absoluta.

AMADO DE DIOS ([email protected])[Escriba texto] Página 11

Autor: Amado Crisógono Castro Chonta Entonces:

2015-2

Para un estado (1) V1  Vo

Para un estado (2)

(273,15  t1 ) 273,15

V2  Vo

273,15 + t1 = T1 V1 

(273,15  t 2 ) 273,15

273,15 + t2 = T2

VoT1 273,15

V2 

VoT 2 273,15

dividiendo las ecuaciones de los dos estados termodinámicos y factorizando tenemos:  V V1 VoT1 / 273 T     1  1  V2 VoT 2 / 273  V2 T2 

…………………………..(9) P1m

Para una misma masa de un gas ideal en un proceso termodinámico a presión constante, a mayor temperatura mayor volumen, entonces el volumen (V) y la temperatura (T) son (V  T)P,m

directamente proporcionales:

Su ecuación de estado termodinámico es:  V   T

(V = KCh T)P,m ó 

 K Ch

..............(10)

P ,m

Donde KCh es la constante de Charles en merito a estas leyes descubiertos y estudiados por el científico Jacques Charles. Ecuación termodinámico para cambios de estado termodinámico: (

V1 V2 Vn   ..............   KCh )P,m. ……...(11) T1 T2 Tn

Representación Gráfica : La ecuación (8) es la ecuación de una recta que corta el eje de Vt = Vo + Vo t ...............................................(8)

volumen: Si

t1= 0

y

Si

Vt1 = Vo

 t2 = -273,15ºC

 Vt2 = 0

Donde Vo es una constante que es igual a 22,414 L por cada molg de Gas Ideal (GI) a la temperatura de 0ºC y presión una atmosfera. V

(Vo) = Tg

Pendiente de la recta

Vo = 22,414L 0

-273,15ºC =0 0ºK

t ºC

AMADO DE DIOS ([email protected])[Escriba texto] Página 12

Autor: Amado Crisógono Castro Chonta

2015-2

Diagrama Nº 5: Comportamiento del GI en el gráfico V vs. t. De la misma manera la representación gráfica de la ecuación de estado de la Ley de Jacques Charles es función de la temperatura absoluta, y es una línea recta ( V = KChT)m,P…………………………(10) Vemos que la ecuación (10) no tiene intercepto, y parte del origen de coordenadas en el diagrama volumen vs temperatura (V vs T). En el diagrama se expresa rectas isóbaras que nos indica valores de presión constante de mayor a menor (Pn>P3>P2>P1) y cada recta es un valor de la constante de Jacques Charles donde KGL1>K GL2>K GL3>K GLn Pn > P3 > P2 > P1

Diagrama Nº 6: Las isóbaras de Jacques Charles V(L) P1 P2 CADA RECTA Pi ES UNA

P3

ISÓBARA Pn

T(K) También se expresa la ecuación de Jacques Charles en función de sus densidades () (

1 V2  )m,P  2 V1

y

(

V2 T2  )  V1 T1 m,P

(

 2 T1  )m,P …... 1 T2

(11)

La densidad de una misma masa de gas a presión constante es inversamente proporcional a su temperatura absoluta. Algunos experimentos que pueden demostrar esta ley son los siguientes: En un tubo de ensayo se deposita un poco de agua y se tapa el tubo con un corcho, luego se empieza a calentar el tubo con un mechero, el gas que había dentro del tubo (el vapor generado por el agua y el aire) empezara a expandirse, tanto que necesita una vía de escape, así que ¡pum! el corcho saldrá volando y el gas ya podrá salir tranquilamente. Para este experimento se necesitara una botella de vidrio, un mechero y un globo de caucho. En la punta de la botella pondremos la boca del globo y luego calentaremos la botella. Después de un buen rato el gas se expenderá hasta inflar el globo de caucho. Un experimento muy contrario a este seria meter un globo inflado totalmente a una nevera, si con el calor los gases se expanden, con el frío pasa todo lo contrario, así que después de esperar unas 3 horas veremos el globo un poco desinflado.

AMADO DE DIOS ([email protected])[Escriba texto] Página 13

Autor: Amado Crisógono Castro Chonta

2015-2

Jacques Alexandre César Charles (Beaugency-sur-Loire, 12 de noviembre de 1746 - 7 de abril de 1823) inventor, científico y matemático francés. Fue el primero en realizar un viaje en globo aerostático, el 27 de agosto de 1783. El 1 de diciembre de ese año, junto con Ainé Roberts, logró elevarse hasta una altura de 550 metros. Inventó varios dispositivos, entre ellos un densímetro (también llamado hidrómetro), aparato que mide la gravedad específica de los líquidos. Cerca del 1787 descubrió la ley de Charles. Su descubrimiento fue previo al de Louis Joseph Gay-Lussac, que publicó en 1802 la ley de expansión de los gases. Charles fue electo en 1793 como miembro de la Académie des Sciences, instituto real de Francia. Fue profesor de física hasta su muerte el 7 de abril de 1823. SEGUNDO CASO: “LEY DE LAS ISÓCORAS” Ley de las ISÓCORAS O ISOMÉTRICAS, expresa que a volumen constante la presión de una determinada masa de un gas ideal aumenta en 1/273,15 = 0,003 66 = , de su presión por cada grado (ºC) de elevación en su temperatura análogo al 1er. Caso Po: presión del gas a 0ºC Pt: presión del gas a una temperatura t (ºC) Pt = Po + Po t  P1 T1      P2 T2 

porque V ,m

Pt = Po (1 + t)

factorizando  273  t  Pt  Po   273 

 P    KGL  T V1m

(P = KGL T )V,m ………(12)

Donde KGL es la constante de Gay-Lussac en merito a estas leyes descubiertos y estudiados por el científico Joseph-Louis Gay-Lussac Para una misma masa de gas ideal, a volumen constante, la presión del gas es directamente proporcional a su temperatura absoluta. Representación gráfica: En esta ley la presión es función de su temperatura absoluta, que es una línea recta. Al analizar Vn > V3 > V2 > V1

Diagrama Nº 7: Las Isométricas de Joseph-Louis Gay-Lussac P (atm)

V1 V2 V3

CADA RECTA Vi ES UNA Vn

ISÓMÉTRICA T(K)

AMADO DE DIOS ([email protected])[Escriba texto] Página 14

Autor: Amado Crisógono Castro Chonta

2015-2

Louis Joseph o Joseph-Louis Gay-Lussac (Saint-Léonard-de-Noblat, Francia, 6 de diciembre de 1778 - París, Francia, 9 de mayo de 1850) fue un químico y físico francés. Es conocido en la actualidad por su contribución a las leyes de los gases. En 1802, GayLussac fue el primero en formular la ley según la cual un gas se expande proporcionalmente a su temperatura (absoluta) si se mantiene constante la presión. Esta ley es conocida en la actualidad como Ley de Charles. Los gases reales no siguen con exactitud las leyes de Gay – Lussac, Charles y de Boyle. Las variaciones son muchos menores cuando el gas real está a alta temperatura y baja presión. Las desviaciones varían de un gas a otro. PROBLEMA Nº 3: El aire contenido en una llanta de automóvil presenta una presión manométrica de 25 lbf/pulg2 medida a 0ºC. ¿Calcular su presión manométrica a 27ºC? 1º Calcular presión y temperatura absoluta en el estado 1: P = Pman + Po

P  presión absoluta

1 atm = 14,7 lbf/pulg2

Pman  presión manométrica

P = (25 + 14,7) lbf/pulg2

Po

P1 = 39,7 lbf/pulg2 absoluta

T1 = 0ºC + 273,15

 presión atmosférica

T1 = 273,15K 2º Calcular la temperatura y presión en el estado 2: T2 = 27ºC + 273,15 = 300,15K Ecuación: P2  39,7

P1 P2  T 1 T2



lb  300,15 K    pu lg 2  273,15 K 

P2 =?

 T  P2  P1  2  ; reemplazando valores:  T1  tenemos que la P2 = 43,6 lb/pulg2 abs

3º Calcular la presión manométrica del aire a 27ºC, estado 2: Pman2 = P2 – Po

entonces Pman2 = (43,6 – 14,7) lb/pulg2

Respuesta: La presión manométrica a 27ºC es Pman2 = 28,9 lb/pulg2 PROBLEMA 4: Si se conserva constante el volumen de un gas calcular la temperatura en grados Celsius cuando se triplica la presión, si la temperatura inicial medida es 50ºC. 1º Calcular la temperatura absoluta en el estado 1: T1 = (50+273,15)K= 323,15 K 2º Calcular la presión en el estado 2: P2 = 3P1

AMADO DE DIOS ([email protected])[Escriba texto] Página 15

Autor: Amado Crisógono Castro Chonta 3º Calcular la temperatura en el estado 2: T2 = 969,45 K

2015-2

T2  323,15 K (

y en grados Celsius es

luego tenemos:

3P ) P

entonces

t(ºC) = 969,45 – 273,15 ;

t2 = 696,3ºC.

Respuesta: La temperatura final es 696,3ºC PROBLEMA 5: Un frasco abierto contiene aire a 27ºC y 1atm, si se calienta hasta 87ºC a la presión constante atmosférica. determinar la fracción del aire que es expulsado. 1º Calcular la temperatura absoluta en el estado inicial (T1) y su volumen V1: . T1 = 27ºC x

1º K + 273,15K; entonces la temperatura 1º C

T1 = 300,15K

P1 = 1 atm V1 = 10 L valor asumido = V 2º Calcular la temperatura absoluta en el estado final (T2) y su volumen final (V2): T2 = (87 + 273,15)K = 360,15K

Calculando el V2 360,15º K

V2 = l0 L 300,15º K  12 L

P2 = 1 atm V2 =?

Calculando la fracción de volumen expulsado : (12  10) L 1  10 L 5

Respuesta: La fracción de aire expulsado es

1 V 5

PROBLEMA 6: Un frasco abierto contiene aire a 20ºC y 1atm; calcular la temperatura a la que debe calentarse el frasco para que expulse 1/5 del aire que contiene, si se mantiene la misma presión. 1º Calcular las propiedades termodinámicas del estado 1: T1 = 20ºC

1K  273,15 K  293,15º K 1º C

P1 = 1 atm

y

V1 = V0

2º Calcular las propiedades termodinámicas del estado 2: T2 =?

 1  Vo  Vo  5  

V2  

P2 = 1 atm

V2 =

6 Vo 5

Ecuación: T2 = T1 (V2/V1)

AMADO DE DIOS ([email protected])[Escriba texto] Página 16

Autor: Amado Crisógono Castro Chonta

2015-2

6 Vo T2 = 293,15 K 5 Vo

T2 = 351,78 K – 273,15 K

RESPUESTA: El frasco debe calentarse a la temperatura

 T2 = 78,63ºC

de 78,6ºC y 1 atm de presión

Problema 7: Una masa de gas a presión constante ocupa un volumen de 20L a una temperatura de 30ºC. Aplicando la Ley de Gay Lussac calcular lo siguiente: a) el volumen que el gas ocuparía si la temperatura se elevara a 150ºC; b) la temperatura en grados Celsius si el volumen cambia hasta medir 5L.

Problema 8: Una masa constante de gas a temperatura constante tiene una presión manométrica de 1atm en un lugar donde la presión atmosférica es 700 mmHg y ocupa un volumen de 50L. Aplicando la ley de Boyle calcular lo siguiente: a) el volumen que el gas ocuparía si la presión absoluta aumentara en 1000 mmHg; b) la presión del gas si el volumen disminuye hasta medir 10L.

&&&&&&&&&&&&&&&&&&&&&&&&&&&&&&&&&&&&&&&&&&&&&

“Yo te ha amado desde toda la eternidad” Jeremías 31,3. “Antes de ser formado en el vientre de tu madre, Yo te conocía, y antes que nacieses, Yo te escogí” Jeremías 1,5 “Te he llamado por tu nombre y tú me perteneces” Isaías 43,1-2 “Tengo tu nombre grabado en la palma de mis manos y no puedo olvidarme de ti” Isaías 49,15-16. “Todo lo mío es tuyo y todo lo tuyo es mío” San Juan 17,10 “Hasta los pelos de tu cabeza los tengo contados” San Mateo 10,30 “Hijo mío, escucha mis palabras para ser sabio” Proverbios 23,19 “Confía en Mí y no te apoyes en tu propia inteligencia” Prov. 3,5 &&&&&&&&&&&&&&&&&&&&&&&&&&&&&&&&&&

AMADO DE DIOS ([email protected])[Escriba texto] Página 17

Autor: Amado Crisógono Castro Chonta

2015-2

2, 1,3 ECUACIÓN DE ESTADO DEL GAS IDEAL

Diagrama Nº 8: Gas Ideal encerrado en un cilindro-émbolo, cuyo volumen puede variar al mover el émbolo. Para una masa constante la ecuación de estado de un gas ideal será: V = f (P, T)…(13) Un estado termodinámico se fija mediante los parámetros P, V, T

Estado 1

Estado 2

P1, V1, T1

P2, V2, T2 ESTADO

ISOTERMA

T = cte

INTERME

P = cte ISÓBARA

Vx, T1, P2 1º Relacionando el estado 1 con el estado intermedio (T1 = cte) Por la ley de Boyle :  P1  P2

Vx, P2 = P1V1  Vx = V1 







………………(13,1)

AMADO DE DIOS ([email protected])[Escriba texto] Página 18

Autor: Amado Crisógono Castro Chonta

2015-2

2º Relacionando el estado intermedio con el estado 2 donde la P = cte  por la ley de Gay-Lussac : (1er.caso)

 T1  T2

 Vx  V2 

V

V x  2 T T 1 2

  

……………………. (13,2)

3º Igualando la ecuación 13,1 con la ecuación 13,2 V1 P1 V T  2 1 P2 T2

ordenando

V1 P1 V P  2 2 = cte. T1 T2

……………….(14)

La ecuación (14) es la 1ª expresión termodinámica de la ecuación de estado del gas ideal. En general:  PV     T 

 constante = n.R……………………. (15) m

Se ha determinado experimentalmente que el valor de la constante de la ecuación (15) es igual al producto del número de moles (n) por la constante universal de los gases ideales (R). Pero por la Ley de Avogadro: cualquier gas ideal que contenga el mismo número de moles (n) a las mismas condiciones de P y T ocupan el mismo volumen (V), es decir volúmenes iguales contienen el mismo número de moles:

( V = KAn)P,T……..(16)

1 mol- moléculas= 1 molg = 6,023 x 1023 moléculas 1 mol-átomo = 1 molg = 6,023 x 1023 átomos Si el volumen total (V) del gas ideal de la ecuación (15) se cambia por su volumen molar (Vm) esta se transforma en la ecuación (17) donde la nueva constante es igual a la constante universal de los gases ideales (R).  PVm     T 

= constante= R  Constante universal de los gases ideales………(17) m

Vm  volumen molar (es decir el volumen de una molg de sustancia) Vm 

V  V  nVm ……. (18) y n

(Vm)C.N = 22,414 L/mol ………….(18,1)

Para “n” moles la ecuación (17) se transforma en la ecuación (15): P ( nVm )  nR T PV  nR T

ó

donde n  número de moles

PV = nRT…………………..(19)

AMADO DE DIOS ([email protected])[Escriba texto] Página 19

Autor: Amado Crisógono Castro Chonta

2015-2

La ecuación (19) es la 2da expresión termodinámica de la ecuación de estado de un gas ideal, que relaciona n, P, V, T para un único estado termodinámico: Conocemos que el número de moles es:

n

 masa m …….(20) M  masamolar

Reemplazando la ecuación (20) en la ecuación (19) obtenemos: PV 

m RT …………..(21) M M 

Para calcular la masa molar (M) de un gas ideal: Para calcular la masa (m) de un gas ideal:

m

mRT ………(21,1) PV

MPV …………………(21,2) RT

Poniendo la ecuación (21) en función de su densidad:  m  RT …..(21,3) donde la densidad es  V

PM  



m  masa …….(22) V  volumen

Reemplazando la ecuación (22) e la ecuación (21,3) obtenemos: PM = RT ideal:

despejando obtenemos la ecuación (23) para calcular la densidad de un gas 

PM ………………….. (23) RT

Para los gases ideales las relaciones

PVm MPV 1 ó 1 RT mRT

ó 1

PV ….(24) nRT

de estos parámetros según la ecuación (24) es 1 y para los gases reales es distinta de la unidad. PVm  Z  donde Z se llama coeficiente de comprensibilidad…..(25) RT

A condiciones normales de presión y temperatura (0ºC y 1 atm) se ha encontrado que una unidad molg de un gas ideal ocupa un volumen de 22,414 L = 22 414mL. 2, 1, 3a: UNIDADES Y DIMENSIONES DE LA CONSTANTE UNIVERSAL DE LOS GASES IDEALES (R) 1º Hallando las dimensiones de

es P 

R

PV nT

...ecuación (19): las dimensiones de la presión

F  FL2 y del volumen es V L3 y del número de moles es A

nmolg y

de la temperatura es T grado, y reemplazando todo en la ecuación (19) obtenemos:

AMADO DE DIOS ([email protected])[Escriba texto] Página 20

Autor: Amado Crisógono Castro Chonta R 

2015-2

( FL2 )( L3 ) ( FL)  …..…..(26), entonces la constante (R) universal mo lg .grado mo lg .grado

de los gases ideales expresa energía del gas ideal por cada unidad de molg y por cada grado de temperatura R 

ENERGIA ……………….(27) mo lg .grado

2º Determinando las unidades de la constante universal de los gases ideales (R): a) Expresando R en atm. L R

A CNPT: P = 1 atm.

(1atm)(22,414 moLlg )

T = 0ºC + 273 = 273,15 K

grado

Las unidades atm.L son unidades

V = 22,414 L

de energía

n = 1 molg

R

(1atm)(22,414 L) (1mo lg)(273,15 K )



b) Expresado R en ergios

R

(1,0133 x106

dina cm 2

R  0,08206

atm.L mo lg.K

………(28)

Tomando como base CNPT :

)(22414cm 3 )

(1mo lg)(273,15 K )

La unidad ergio es unidad de

P  1 atm = 1,0133 x 106 V  22,414L x

dina cm 2

103 cm 3 1L

V = 22 414cm3

energía.

n = 1 molg Entonces el valor de (R) es:

R = 8,314 x 107 erg/molg.K …………(29)

T = ºC + 273,15 = 273,15 K c) Expresando R en el SI

Tomando como base CNPT 1 julio = 107erg

R  8,314 x10 7

erg mo lg .K

 1 julio    entonces 7  10 erg 

R = 8,314 julio /molg.K ….(30)

d) Expresando R en calorías : tomando como base CNPT Conocemos que 1 cal = 4,184 julio entonces

AMADO DE DIOS ([email protected])[Escriba texto] Página 21

Autor: Amado Crisógono Castro Chonta R  8,314

2015-2

julio 1cal x factorizando mo lg .K 4,184 julio

R  1,987

cal cal  1,99 … mo lg .K mo lg .K

(31) Para muchos cálculos de ingeniería se utiliza el valor de R= 2,00cal/molg.K....(31,1)

PROBLEMA 9: Cierta masa de gas ocupa 200 litros a 95ºC y 782 mmHg ¿Cuál será el volumen ocupado por dicha masa de gas a 65ºC y 815mmHg?

Solución del problema: 1º Las propiedades de P,V,T en los estados 1 y 2 son: Estado 1

Estado 2

P1 = 782 mmHg

P2 = 815mmHg

T1 = (95 + 273,15)K

T2 = (65 + 273,15)K

T1 = 368,15K

= 338,15K

V1 = 200L

V2 = ¿?

2º Utilizando la ecuación (14) para calcular el volumen en el estado 2: P1V1 PV  2 2 ….(14) T1 T2

despejando

 T  PV V2   2  1 1  P2  T1

reemplazando valores

tenemos: V2 

338,15 K 782mmHgx 200 L x entonces el volumen 2 es: V2 = 176,3 L 815mmHg 368,15 K

Respuesta: el volumen final ocupado por el gas es 176,3L PROBLEMA 10: Dentro de un cilindro-pistón está encerrado 1,00 m3 de aire a la presión de 5 atm y a 20C. ¿calcular: a) La presión del aire interior, si manteniendo constante la posición del pistón, se calienta el aire hasta 120ºC? y b) La presión del aire interior, si luego de haber calentado el aire hasta 120ºC, se le deja expandir isotérmicamente hasta ocupar un volumen de 5,00m3? Respuesta a) 6,70 atm

b) 1,34atm

PROBLEMA 11: Una masa gaseosa pasa de 20ºC a 150ºC, ocupando a esta segunda temperatura y a la presión de 600 mmHg un volumen doble del primitivo. ¿Calcular la presión inicial?

AMADO DE DIOS ([email protected])[Escriba texto] Página 22

Autor: Amado Crisógono Castro Chonta

2015-2

Solución del problema: 1º Las propiedades de P,V,T en los estados inicial (1) y final (2) son: Estado inicial (1)

Estado final (2)

T1 = (20 + 273,15)K

T2 = (150 + 273,15)K

= 293,15K

= 423,15K

Sea volumen inicial V1

P2 = 600mmHg

P1 = ¿?

Entonces el V2 = 2V1

2º Utilizando la ecuación (14) para calcular la presión inicial: P1V1 PV  2 2 T1 T2

….(14) despejando P1  P2 x

T1 V2 x T2 V1

remplazando valores

tenemos: P1  600mmHgx

293,15 K 2V 1 x entonces P1 = 831,3 mmHg ó 831,3 torr 423,15 K V1

Respuesta: La presión inicial es 831,3 mmHg ó 831,3 torr ¿Resuelva este problema por un segundo método? PROBLEMA 12: Un cilindro contiene 24kg de cloro líquido. ¿Calcular el volumen en (m3) que el cloro ocuparía si fuera puesto al estado gaseoso a condiciones normales de presión y temperatura (CNPT)?

Solución del problema: 1º Las propiedades de P,V,T del gas cloro a CNPT: El cloro gas es diatómico Cl2, asumiendo comportamiento ideal. P = 1 atm

V = ¿?

1mol Cl2 = 22,414L = 71,0 g

T = 0ºC + 273 = 273,15K

m = 24kg = 24 000g

Masa atómico del cloro es 35,5u entonces la masa molar del cloro es: M = 71,0g/mol. 2º Utilizando la ecuación (21) para calcular el volumen del cloro en el estado gaseoso: PV 

m m RT RT ……(21) despejando el volumen tenemos: V  x M M P

1000 g L.atm )(0,08205 )(273,15K ) 1kg mo lg .K V  g (71,0 ) (1atm) mo lg 24kg (

simplificando el volumen es

1m 3 ) 1000 L

V=7 576L transformando el volumen de L a m3 tenemos: V  7576 L(

AMADO DE DIOS ([email protected])[Escriba texto] Página 23

Autor: Amado Crisógono Castro Chonta

V = 7,576 m3

2015-2

Respuesta el cloro ocuparía un volumen de 7,576 m3

¿Resuelva este problema por un segundo método? 2, 1, 3b: DENSIDAD Y DENSIDAD MOLAR DE LOS GASES: Densidad es el cociente indicado de la masa (m) de una sustancia por su volumen(V): Sea   densidad

que se expresa por

Las unidades de la densidad son:  



m  masa ……………(32) V  volumen

g g kg kg lb    3  ...(32,1) cm3 mL L m pie 3

La densidad molar es el cociente indicado de la densidad de una sustancia por su masa molar: m 

 densidad molar ……(33) entonces al desarrollar M

m 

m  m 1   MV  M  V

Obtenemos que la densidad molar ecuación (33) equivale a la ecuación (34) que es el cociente indicado del número de moles (n) de una sustancia por su volumen (V): m 

m 

n  númerodemo les ………...(34) entonces sus unidades son:  volumen V

mo lg  n =  M V cm3



mo lg ml



mo lg mo lg  3 ………(34,1) l m

2, 1, 3c: DENSIDAD RELATIVA DE LOS GASES IDEALES: Es la relación existente entre la densidad de un gas y la densidad de un gas de referencia; en este caso es generalmente el aire a las condiciones normales de presión y

aireCN = 1,289g/l = 1,289kg/m3 …………..(32,2)

temperatura (CNPT):

Cuando se comparan dos densidades se deben hacer a las mismas condiciones de presión y temperatura:  A( B ) 

Donde:

A B

entonces

r 

(  A ) P .T (  aire ) P.T

………(35)

r es densidad relativa; A es la densidad del gas problema; aire es la

densidad del aire y B es la densidad del gas de referencia. Aplicando la ecuación (32) a las sustancias “A” y “B”: (  A ) P1T  m A / V A

y

(  B ) P1T  m B / V B ………………(32,3)

AMADO DE DIOS ([email protected])[Escriba texto] Página 24

Autor: Amado Crisógono Castro Chonta

2015-2

Dividiendo las ecuaciones (32,3) y aplicando la ley de AVOGADRO: “Los gases ideales a las mismas condiciones de P y T a igual número de moles (nA= nB) sus volúmenes son iguales (VA = VB ) ………..(36)

r 

mA / VA m entonces  simplificando r  A ………………(35,1) m B / VB mB

Conocemos que la masa de una sustancia es: mA = nAMA y mB=nBMB reemplazando en la ecuación (35,1) se obtiene: ( r ) P , T 

nAM A nB M B

y simplificando si VA = VB  nA = nB entonces ( r ) P ,T 

MA …. MB

(37) La densidad relativa de un gas ideal a las mismas condiciones de presión y temperatura es una simple relación de sus masas molares.

PROBLEMA 13: ¿Hallar la densidad del gas: a) metano (CH 4); b) etano (CH3CH3) ; c) O2; y d) etino; en g/l a condiciones normales de presión y temperatura (CNPT)?

Solución del problema 13a: 1º Las propiedades del metano a CNPT son: Asumiendo que el gas metano (CH4) se comporta como un GI a CNPT P = 1 atm

Vm = 22,414L/mol T = 273,15K

M = 16g/molg

2º Utilizando la ecuación (23) para calcular la densidad del gas metano: (1atm)(16 g / mo lg)  PM L.atm  ……(23) reemplazando valores (0,08205 )(273,15 K ) RT mo lg .K

Respuesta: La densidad del gas metano a CNPT es

= 0,714 g/L

PROBLEMA 14: Se tiene encerrado en un cilindro gas anhídrido sulfuroso (SO2) Calcular: a) Su densidad (g/l) a 27ºC y 2 atm y b) Su densidad relativa en el Sistema Internacional (SI) y en el Sistema de Ingeniería Americano (SIA)

Solución del problema: 1º Las propiedades de P,V,T del SO2 son: T = (27 + 273,15)K T = 300,15K

P = 2 atm MSO2 = 64,0 g/molg

L.atm

R = 0,082 05 mo lg .K Maire = 28,9 g/molg

AMADO DE DIOS ([email protected])[Escriba texto] Página 25

Autor: Amado Crisógono Castro Chonta

2015-2

2º Utilizando la ecuación (23) para calcular la densidad del SO2: 



PM …(23) RT

reemplazando valores

( 2atm)(64,0 g / mo lg) L.atm (0,08205 )(300,15 K ) mo lg .K

Respuesta “a”: La densidad del SO2 es:  = 5,200 g/L 3º Utilizando la ecuación (37) para calcular la densidad relativa del SO2:

r 

MA …..(37) MB

64,0 g / mo lg

reemplazando valores su r = 28,9 g / mo lg

Respuesta “b” su densidad relativa es

r = 2,21

La densidad relativa es una propiedad termodinámica del estado de una sustancia y es adimensional. PROBLEMA 15: La masa molar de un gas “A” es la tercera parte de aquel de otro gas “B”. A igual V y T la densidad del gas “A” es el doble de aquella del gas “B”. ¿Si dos bulbos idénticos contienen separadamente a los gases “A” y “B”, calcular la relación de la presión del gas A con respecto a la del gas B?

&&&&&&&&&&&YO TE AMO MUCHO&&&&&&&&&&&&& “Mi amor nunca se apartará de tu lado” Isaías 54,10 “Yo nunca te dejaré, ni te abandonaré” Hebreos 13,5 “No tengas miedo, porque Yo estoy contigo” Isaías 43,5 “No tengas miedo, solamente confía en mi” San Marcos 5,36 “Dame hijo mío tu corazón, ámame, y que tus ojos hallen deleite en mis caminos” Proverbios 23,26 “Tú eres precioso a mis ojos, de gran estima y Yo te amo mucho” Isaías 43,4 &&&&&&&&&&&&&&&&&&&&&&&&&&&&&&&&&&&& AMADO DE DIOS ([email protected])[Escriba texto] Página 26

Autor: Amado Crisógono Castro Chonta

2015-2

2, 1, 4 MEZCLAS DE GASES IDEALES: 2, 1, 4a: LEY DE LAS PRESIONES PARCIALES DE DALTON:

Diagrama Nº 9: De las presiones parciales de Dalton. La presión total (P) de una mezcla de gases es igual a la suma de las presiones parciales (Pi) de los componentes de una mezcla gaseosa confinados en un mismo volumen (V=cte.) y a una misma temperatura (T=cte.) en todo el proceso termodinámico. Sea P = Presión total y Pi es la presión parcial del componente “i” de la mezcla; entonces la presión total de una mezcla gaseosa es: m

( P   Pi  Pa  Pb  Pc  ...  Pm ) V,T…....(38) i a

Se llama presión parcial (Pi) de cada gas en una mezcla gaseosa a la presión que dicho gas ejercerá si ocupará solo el volumen total de la mezcla y a la misma temperatura: Se tiene 3 gases

Pa V = na RT

ecuación de estado del gas (a)…. (19a)

Pb V = nb RT

ecuación de estado del gas (b)…. (19b)

Pc V = nc RT

Ecuación de estado del gas (c)…. (19c)

Sumando las tres ecuaciones tenemos: (Pa + Pb + Pc) V = (na + nb + nc) RT ……(39) Si la presión total de la mezcla es:

P = Pa + Pb + Pc………..(38,1)

Y también el número de moles totales de la mezcla es: n = na+nb+nc…(40)

AMADO DE DIOS ([email protected])[Escriba texto] Página 27

Autor: Amado Crisógono Castro Chonta

2015-2

Entonces reemplazando las ecuaciones (38,1) y (40) en la ecuación (39) obtenemos la ecuación (41):

PV = nRT….(41)

Esta ecuación es idéntico a la ecuación (19)

por lo tanto la misma ecuación se puede utilizar tanto para mezcla de gases ideales y gases ideales puros. Ahora dividiendo cada ecuación (19a), (19b) y (19c) entre la ecuación (41) se obtiene: Pa  na  n    Pa = xaP donde a  xa se llama fracción molar del componente “a” P  n n de la mezcla gaseosa igual a su fracción de presión, y para el componente “b” será Pb = xbP donde xb es fracción molar del componente “b” de la mezcla igual a su fracción de presión, y para el componente “c” será Pc = xcP donde xc es la fracción molar del componente “c” de la mezcla igual a su fracción de presión. En general entonces la fracción molar del componente “i” de la mezcla es igual a su fracción de presión; y la presión parcial de un componente “i” de la mezcla es

pi = xi P………(42)

La fracción de presión parcial ejercida por cada componente de una mezcla gaseosa es directamente proporcional a su fracción molar (xi), llamada también concentración molar, en dicha mezcla:

pi ni  xi = …..(42a) y el porcentaje de su concentración p n

molar de un componente de la mezcla gaseosa es equivalente a su porcentaje de presión:  pi   p

 

 100  xi (100) =

ni (100)………… (42b) n

Dividiendo la ecuación (39) entre la ecuación (41) y simplificando se obtiene: p a  p b  p c n a  nb  nc  p n

= 1 …………..(43)

que nos dice que la sumatoria de las

fracciones de presión de todos los componentes de una mezcla es igual a la unidad: p a pb pc p a  pb  pc p     = p p p p p

1…….(43a)

Y la sumatoria de las fracciones molares de todos los componentes de una mezcla es = 1 n a nb n c n a  nb  n c n      1 …………(43b) n n n n n

John Dalton (Eaglesfield, Cumberland (Reino Unido), 6 de septiembre de 1766 Manchester, 27 de julio de 1844), fue un naturalista, químico y matemático,

AMADO DE DIOS ([email protected])[Escriba texto] Página 28

Autor: Amado Crisógono Castro Chonta

2015-2

meteorólogo británico. En 1801 enunció la ley de las presiones parciales y la de las proporciones múltiples.3 En 1808 expuso la teoría atómica en la que se basa la ciencia física moderna. Demuestra que la materia se compone de partículas indivisibles llamadas átomos. También ideó una escala de símbolos químicos, que serán luego reemplazadas por la escala de Berzelius.4 En 1826 se le concedió la Medalla de Oro de la Royal Society de Londres, así como de la Academia Francesa de las Ciencias. Falleció en Manchester en 1844, a la edad de 78 años. Más de 40.000 personas acudieron al funeral para presentar sus respetos al científico5 6

2, 1, 4b: LEY DE LOS VOLUMENES PARCIALES DE AMAGAT: El volumen total (V) ocupado por una mezcla gaseosa es igual a la suma de los volúmenes parciales (Vi) de sus gases componentes que se mantienen a temperatura constante (T=cte.) y presión constante (P=cte.) en todo el proceso termodinámico. El volumen parcial de un componente “i” de una mezcla gaseosa (Vi), es el volumen que ocupará aquel componente gaseoso si el solo estuviera presente a la misma presión y temperatura que tiene la mezcla: Va , Vb, Vc

son los volúmenes parciales de los gases componentes de una mezcla a

las mismas condiciones de P y T entonces el volumen total de una mezcla de gases

( V = Va + Vb + Vc )P,T ………(44)

ideales es:

Se tiene tres gases ideales que forman una mezcla: Va P = na RT ecuación de estado del gas ideal (a)…….(19d) Vb P = nb RT ecuación de estado del gas ideal (b)…….(19e) Vc P = nc RT ecuación de estado del gas ideal (c)…….(19f) Sumando las tres ecuaciones se obtiene: (Va + Vb + Vc) P = (na + nb + nc) RT ….(45) Si el volumen total de la mezcla es:

V = Va + Vb + Vc …………………….(44a)

Y también el número de moles totales de la mezcla es: n = na + nb + nc ……(40) Entonces reemplazando las ecuaciones (44a) y (40) en la ecuación (45) obtenemos la ecuación: PV

= nRT…(41).

Esta ecuación es idéntico a la ecuación (19)

por lo tanto se puede utilizar tanto para mezcla de gases ideales y gases ideales puros. Ahora dividiendo cada ecuación (19d), (19e) y (19f) entre la ecuación (41) se obtiene

AMADO DE DIOS ([email protected])[Escriba texto] Página 29

Autor: Amado Crisógono Castro Chonta las ecuaciones: za =

2015-2

Va n a  = xa donde la fracción volumétrica del componente “a” V n

de la mezcla es igual a su fracción molar y V a = xaV; y zb =

Vb nb  = xb V n

donde la

fracción volumétrica del componente “b” de la mezcla es igual a su fracción molar y Vb = xbV; y zc =

Vc nc  = xc donde la fracción volumétrica del componente “c” de la V n

mezcla es igual a su fracción molar y Vc = xcV. En general el volumen parcial de un componente “i” de la mezcla es igual al producto de su fracción molar por el volumen total de la mezcla:

Vi = xi V…(46)

y

La fracción del volumen parcial (zi) ejercida por cada componente de una mezcla gaseosa es directamente proporcional a su fracción molar (x i), o concentración molar, en dicha mezcla: zi =

pi ni  xi = …..(47) y el porcentaje de la concentración molar de p n

un componente de la mezcla gaseosa es equivalente al porcentaje de su fracción volumétrica:

 Vi  ni  100  xi (100) = (100) entonces % zi  % xi ………… n  V

100zi= 

(47a) PROBLEMA 16: ¿Calcular la composición molar porcentual del oxigeno y del nitrógeno? 1º La composición volumétrica aproximada del aire es: Oxígeno 21% y nitrógeno 79% en volumen. 2º Hacer una Tabla para calcular composición molar Componentes Volumen

% volumen

=

% molar

O2

21L

21 x100  21% 100

21%

N2

79L

79 x100  79% 100

79%

(aire)

100 L

Aplicando la ley de Amagat: %zi = %xi

Respuesta: La composición molar porcentual del oxígeno es 21% y del nitrógeno79%.

AMADO DE DIOS ([email protected])[Escriba texto] Página 30

Autor: Amado Crisógono Castro Chonta

2015-2

PROBLEMA 17: La composición volumétrica aproximada del aire seco es O2=21,0% y N2 = 79,0%. ¿Hallar su composición en masa y nasa molar de la mezcla? 1º Para determinar la composición volumétrica, %zi = 100(Vi/V), de una mezcla: Mezcla gaseosa

Gas

Vi

a

Va

100(Va/V)

b

Vb

100(Vb/V)

c

Vc

100(Vc/V)

Aire

%zi = 100(Vi/V)

V = Va + Vb + Vc

100

2º Para determinar la composición molar ,%xi = 100(ni/n), de una mezcla:

Mezcla gaseosa

Gas

ni

a

na

100(na/n)

b

nb

100(nb/n)

c

nc

100(nc/n)

Aire

%xi = 100(mi/m)

n = na + nb + nc

100

3º Para determinar la composición en masa, %yi = 100(mi/m), de una mezcla:

Mezcla gaseosa

Gas

mi

100 yi = %yi

a

ma

100 (ma/m)

b

mb

100 (mb/m)

c

mc

100 (mc/n)

Aire

m = m a + mb + mc

100

4º Tomando como base de cálculo 100 moles de aire: Gas O2

%xi=%zi 21

xi 0,21

ni(mol) 21 mol

Mi 32g/mol

672g

N2

79

0,79

79 mol

28g/mol

2 212g

mi

%yi= 100(mi/m) 672 x100  23,3 2884 2212 x100  76,7 2884

Aire 100% n=100 mol m=2884g 100%  Para calcular el número de moles del oxígeno se multiplica las 100 mol de aire por su porcentaje molar n02 = xO2(100) = (0,21)100 mol

entonces

n02 = 21

mol O2 

Para calcular la masa del oxígeno se multiplica su número de moles por su masa molar: mO2= ni. Mi = 21 mol (32g/mol) entonces la masa es

mO2 = 672g O2

AMADO DE DIOS ([email protected])[Escriba texto] Página 31

Autor: Amado Crisógono Castro Chonta

2015-2

 Para calcular la masa total de la mezcla se suman la masa del oxígeno con la masa del nitrógeno: m = mO2 + mN2 = 672g + 2 212g

entonces la masa total

es m = 2 884g

 Para calcular la composición en masa del oxígeno se multiplica por 100 la fracción másica del oxígeno: %xO2 = 100(mO2/m) = 100(672g/2 212g) entonces la composición en masa del oxígeno es: 

%xO2 = 23,3 %

Respuesta

Calculando la masa molar de la mezcla (M) se divide la masa total de la mezcla (m) entre el número de moles totales de la mezcla (n):M = m/n=2 884g/100mol entonces la masa molar de la mezcla es: M = 28,84g/mol

:Respuesta

PROBLEMA 18: En la fabricación del ácido sulfúrico a partir de la tostación de sulfuros metálicos, el gas de tostación sale a 155ºC y 550 mmHg y contiene 8,1% en volumen de SO2. ¿Calcular la masa en gramos de SO2 contenida por m3 de dicho gas de tostación? Solución del problema: 1º Las propiedades de P,V,T del gas de tostación en la salida son:

Sea el volumen del gas de tostación 1m3 = 1000 L Asumir que el SO2 se comporta como un GI T = (155 + 273,15)K = 428,15K

P = 550 mmHg

%yi = 8,1%  porcentaje en volumen de SO2

MSO2 = 64,0g/molg

2º Calcular el volumen en litros del SO2 en el gas de tostación: VSO2 = 1000L(0, 081) = 81 L de SO2 3º Para calcular la masa en granos de SO2: se aplica la ecuación (21,2) m

PVM …..(21,2) reemplazando valores RT

m 

 

(550mmHg )(81L)(64,0 g / mo lg) L.atm  760mmHg  (428,15º K )( 0,08205 ) mo lg .K  1atm

m = 106,8 g de SO2

Respuesta: Hay 107 gramos de SO2 por cada m3 de gas de tostación. Otro método: 1º Es calcular el volumen del SO2 a CNPT sea (Va), usando la ecuación

AMADO DE DIOS ([email protected])[Escriba texto] Página 32

Autor: Amado Crisógono Castro Chonta

2015-2  P1  Pa

PV P1V1  a a …..(14) despejando T1 Ta

Va  V1 

  Ta     T1   

 550mmHg   273,15K      760mmHg   428,15K 

Reemplazando valores Va  81L

Entonces el volumen del SO2 a CNPT es Va = 37,397L 2º Calcular la masa del SO2 por cada m3 del gas de tostación: A CNPT: 1 molg SO2 = 64,0g = 22,414 L m

37, 397L entonces m 

(37,397 L)(64,0 g ) ( 22,414 L)

m = 106,8 g de SO2 Se demuestra que por los dos métodos se llega al mismo valor de 106,8 g de SO 2 por cada m3 del gas de tostación.

Para resolver un problema hay muchos caminos, entonces tenemos que buscar el camino más corto y simple. PROBLEMA 19: Un hidrocarburo tiene una densidad al estado de vapor igual a 2,55 g/L a 100ºC y 760mmHg. El análisis indica que esta sustancia contiene un átomo de hidrógeno por cada átomo de carbono. ¿Cuál es la fórmula molecular de este compuesto? Solución del problema: 1º Los datos del problema son: Densidad del vapor  = 2,55 g/L

Temperatura

Presión P = 760 mmHg = 1 atm.

Relación carbono-hidrógeno

T = (100 + 273,15)K = 373,15K CnHn y su

masa molar empírica es ME = 13,0 g/mol 2º Calcular la masa molar del compuesto: Se usará la ecuación (21,3) M 

mRT …..(21,3); introduciendo el parámetro densidad y despejando masa VP

molar M 

RT ….(21,4); reemplazando valores: P L.atm )(373,15 K ) mo lg .K 1atm

( 2,55 g / L)(0,08205 M 

entonces

la masa molar

verdadera es: M = 78, g/molg 3º Calcular la fórmula molécula del hidrocarburo:

AMADO DE DIOS ([email protected])[Escriba texto] Página 33

Autor: Amado Crisógono Castro Chonta

2015-2

Se divide la fórmula molar verdadera con respecto a la fórmula empírica para obtener cuántas (n) veces la fórmula verdadera contiene a la fórmula empírica: 78 g / mo lg  6CH . Luego la fórmula verdadera es 13 g / mo lg

6CH = C6H6

El benceno.

Respuesta: El hidrocarburo es el benceno C6H6. PROBLEMA 20: Un bulbo de vidrio de una capacidad de 200ml, al llenarlo con cierto gas a 25ºC y 740mmHg tiene una masa de 27,611g. Al reducir la presencia del gas a 150mmHg, la masa del bulbo con este resto de gas fue de 27,411g. ¿Calcular la masa molar del gas y la masa del bulbo de vidrio? SOLUCIÓN: 1º Los datos del problema son: V = 200mL T = (25 + 273,15)K = 298,15K La masa del estado inicial: m1 = 27,611g

P1 = 740 mmHg

P2 = 150 mmHg

y la masa del estado final: m2 = 27,411g

M, V y T son comunes para los dos estados

M = ¿?

2º Calcular la masa molar de la sustancia en función de la cantidad de gas desalojada:

P1V 

m1 RT …….Estado 1 M

P2V 

m2 RT …….Estado 2 M

Restando las dos ecuaciones para eliminar la masa del bulbo de vidrio tenemos: V(P1 – P2) = (m1 – m2)

RT ...(21,6) entonces M

M = (m 1 – m2)

RT … V(P1 - P2)

(21,7) Calculando la presión del gas desalojado: P = P1 – P2 = (740 – 150)mmHg = 590mmHg Calculando la masa del gas desalojado m=(m1 – m2)=(27.611 – 27.411)g = 0,200g Reemplazando valores en la ecuación (21,7) tenemos:  L.atm  (0,200 g ) 0,08205  ( 298,15º K ) mo lg .K   M  ;   1atm  590mmHg ( 760 mmHg )  (0,2l )  

La masa molar es M =

31,5g/ml

AMADO DE DIOS ([email protected])[Escriba texto] Página 34

Autor: Amado Crisógono Castro Chonta

2015-2

3º Calcular la masa del gas en el estado inicial: Usar la ecuación P1VM ……….(21,2) …………Reemplazando valores tenemos: RT

mG1 

mG1  

 

(740mmHg )(0,2 L)(31,5 g / mo lg) L.atm  760mmHg 0,08205  ( 298,15º K )( ) mo lg .K  1atm

La masa del gas en el estado inicial es: mG1 = 0,251 g 4º Calcular la masa del bulbo de vidrio: mbv = m1 – mG1 = (27,611 – 0,251) g

Respuesta: La masa del bulbo de vidrio es mbv = 27,36 g PROBLEMA 21: Por electrolisis de una salmuera se obtiene en el ánodo una mezcla de gases que tiene la siguiente composición en masa: Cl2 = 70,0%

Br2 = 25,0%

O2 = 5,0%

¿Hallar la: a) Composición de la mezcla gaseosa en volumen? b) Densidad de la mezcla en g/L a 270ºC y 1 atm? c) Densidad relativa (o gravedad específica) de la mezcla? d) Masa molar de la mezcla? e) Composición molar de la mezcla gaseosa?

Solución del problema: 1º Los datos el problema son: Composición en masa de la mezcla gaseosa es: Cl2 = 71,0%

Br2 = 24,0%

O2 = 5,0%

Temperatura de cada componente y de la mezcla es: T = (270 + 273,15K = 543,15K La presión de la mezcla es: 1 atm.

 = ¿?

r = ¿?

2º Tomar como base de cálculo 100g de mezcla gaseosa: 3º Hacer una tabla para calcular las composiciones de la mezcla: Gas Cl2

%yi 71,0

yi 0,710

mi(g) 71,0g

Mi 71g/mol

1,00molg

%x i = %zi 76,34

Bn2 O2

24,0 5,0

0,240 0,050

24,0g 5,0g

160g/mol 32g/mol

0,15molg 0,16mol

11,45 12,21

ni

AMADO DE DIOS ([email protected])[Escriba texto] Página 35

Autor: Amado Crisógono Castro Chonta

2015-2

Mezcla 100% 1,0 m=100g n=1,31molg 100%  Para calcular la masa del cloro se multiplica las 100g de mezcla por su fracción másica mCl2 = yCl2(100) = (0,71)100g entonces m0Cl2 = 71g Cl2 

Para calcular las moles de cloro se divide la masa del cloro entre su masa molar: nCl2= mCl2/MCl2 = 71,0g/ (71g/mol)

entonces el número de moles es n Cl2 =

1,00molg Cl2

 Para calcular las moles totales de la mezcla se suman las moles de los componentes de la mezcla: 0,16molg

n = nCl2 + nBr2 + nO2= 1,00molg + 0,15molg +

entonces las moles totales de la mezcla es n = 1,31molg

 Para calcular la composición molar del cloro se divide las moles de cloro por las moles totales de la mezcla y se multiplica por 100: %xi = 100( nCL2/ n) reemplazando valores tenemos: %xi=100(1,00molg/1,31molg) = 76,34%

Respuesta (e).  Para calcular la composición volumétrica: Por la ley de Amagat la composición molar de un componente de la mezcla es igual a su composición volumétrica: entonces para el gas cloro tenemos %xCl2 = % zCl2 =76,34% :Respuesta (a) 

Calculando la masa molar de la mezcla (M) se divide la masa total de la mezcla (m) entre las moles totales de la mezcla (n):M = m/n=100g/1,31molg entonces la masa molar de la mezcla es: M = 76,34g/mol



:Respuesta (d)

Calcular la densidad de la mezcla en g/l: aplicando la ecuación la ecuación (23) (100 g / 1,31mo lg)(1atm) m  MP  L.atm   …(23); reemplazando valores  0,08205   543,15 K  RT mo lg .K  

La densidad de la mezcla es: 

 = 1,713 g/L

Para calcular la densidad relativa de la mezcla gaseosa se debe transformar la densidad de la mezcla a CNPT y aplicando las ecuaciones: (

CNPT T1  1 TCNPT

 543,15 K    273,15 K 

)m,P...(11 )reemplazando valores CNPT  (1,713g / l ) densidad de la mezcla a CNPT es:

entonces la

CNPT = 3,406g/l

AMADO DE DIOS ([email protected])[Escriba texto] Página 36

Autor: Amado Crisógono Castro Chonta

2015-2

Y para calcular la densidad relativa se usa la ecuación:  r  3,406 g / L

Reemplazando valores r == 1,289 g / l

(  A ) P.T ……(35) (  aire ) P.T

entonces rRespuesta

(c) PROBLEMA 22: Un bulbo con llave y sin aire, se llena con gas CO 2 a la presión del ambiente y por diferencial de pesada se encuentra que contiene 4,40g del gas a una determinada temperatura de T(K). Luego se coloca el bulbo en un baño de temperatura constante, que marca una temperatura de 30ºC más alta que la primera. Allí, estando la salida de la llave fuera del agua, se abre ésta hasta que la presión del gas CO2 retoma al valor original. Entonces el bulbo contiene 3,96g de CO2 ¿Calcular el valor de la temperatura T(K) inicial?

Solución del problema: 1º Los datos del problema en los estados inicial y final son: Asumiendo que el CO2 se comporta como un GI ESTADO INICIAL

ESTADO FINAL

m1 = 4,40g CO2

m2 = 3.96g CO2

V = cte.

V = cte

T1 = T(K)

T2 = (T + 30)K

P1 = P

MCO2 = 44 g/molg

PV 

m1 RT M

PV 

P2= P1 = P

m2 R(T  30) M

2º Calcular el valor de la temperatura: 

Restando las dos ecuaciones de estado:



Desarrollando y despejando la temperatura: (m1 – m2) T = 30m2





m1T = m2 (T + 30) m1T = m2 T + (m2) 30 T

30m2 m1  m2

Reemplazando valores la temperatura inicial es: T 

30 K (3,96 g ) T= 270K ( 4,40  3,96) g

Respuesta la temperatura inicial es 270K PROBLEMA 23: A) ¿Calcular la presión total ejercida por una mezcla gaseosa que se compone de 320g de O2, 1120g de N2 y 20g de H2; Si dicha mezcla está contenida en

AMADO DE DIOS ([email protected])[Escriba texto] Página 37

Autor: Amado Crisógono Castro Chonta

2015-2

un cilindro de 1m3 de capacidad a la temperatura de 17ºC? B) ¿Calcular la fracción molar y presión parcial de cada componente? Solución del problema: 1º Datos del problema: V = 1m3 = 1000 L mO2 = 320g

T = 17 + 273,15 = 290,15K mN2 = 1120g

mH2 = 20g

2º Calculo de las moles totales de la mezcla: nO2 = 320 gx

n = nO2 + nN2 + nH2

1mo lg  10mo lg O2 32 g

1mo lg

nN2 = 1120 gx 28 g  40mo lg N2

1mo lg )  10mo lg H 2 nH2 = 20 g ( 2g

n = 10 + 40 + 10 = 60molg

3º Calcular la presión total de la mezcla: Aplicar la ecuación (19)……..PV = nRT 

 0, 08205

P  (60mo lg) 

L.atm   (290,15 K ) mo lg.K 1000 L

entonces la presión es P = 1,429 atm

4º Calcular la fracción molar y presión parcial de cada componente: Respuesta: Composición molar

Presión parcial de cada componente:

10 x100  16, 67% 60

PO2 = XO2 (1,429 atm) = 0,238atm

X O2 

X N2 

40 x100  66, 66% 60

PN2 = XN2 (1,429 atm) = 0,953atm

XH2 

10 x100  16, 67% 60

PH2 = XH2 (1,429 atm) = 0,238atm

100,00%

Presión total de la mezcla

1,429atm

PROBLEMA 24: En un tubo de vidrio lleno de mercurio, invertido sobre una cuba de Hg, se ha recogido por desplazamiento de este, que llenaba el tubo un volumen de 700cc CO2 en las condiciones siguientes: Temperatura 30ºC, presión atmosférica 740mmHg y la altura sobre el nivel del Hg dentro del tubo fue de 14cm sobre el nivel del mercurio en la cuba. ¿Calcular a) La presión del gas

b) y la masa del gas

recogido? Solución del problema: 1º Datos del problema: T = (30 +273,15)K = 303,15K

Ptotal = Patm = 740 mmHg

Pman = 100mmHg

AMADO DE DIOS ([email protected])[Escriba texto] Página 38

Autor: Amado Crisógono Castro Chonta

2015-2

2º Calcular la presión del gas; Pgas = Ptotal – Pman = (740 – 140)mmHg = 600mmHg 3º Calcular el volumen el gas a CNPT: aplicando la ecuación (14) V1 P1 V P V PT 2  2 2 = cte. Entonces V2 = 1 1 (14) reemplazando valores T1 T2 T 1P 2

700mL(6OOmmHg )(273,15)  V 2 entonces V2 = 498 mL = 0,498L de CO2 (760mmHg )(303,15K ) 4º Calcular la masa del gas CO2 recogido:  1mo lg   44 g     22, 414 L  1mo lg

mCO2  0, 498LCO 2 

entonces

mCO2 = 0,978g

Respuesta: la presión del CO2 es 600mmHg y su masa es 0,978g PROBLEMA 25: Se quiere preparar una mezcla gaseosa que contenga 10% molar de butano (C4H10) y 90% molar de neón. En un cilindro sin aire se introduce butano gaseoso hasta que su presión es de 1 atm. Luego se inyecta dentro del cilindro gas neón comprimido, para formar una mezcla de la composición deseada. El volumen del cilindro es de 20 L y la operación se realiza a 25ºC. ¿Calcular: a) Los moles de butano presente? Rpta: 0,82 moles b) Los moles de neón necesario? Rpta: 7,38 moles c) La presión total de la mezcla gaseosa final? Rpta: 10 atm. d) Composición másica y volumétrica? Otros datos:

Masa atómico del: neón 20,2u; hidrógeno 1,01u; carbono 12,01u

PROBLEMA 26: Un gas consiste de una mezcla de etano y butano. Un balón de 200ml se llena con este gas a la presión de 750torr y a 20ºC. Por diferencia de masa la masa del gas es de 0,384 6g. ¿Calcular las composiciones: a) molar; b) volumétrico; y c) masa de la mezcla?

PROBLEMA 27: Un cilindro de 10 L de capacidad contiene un gas a 27ºC y 5,00 atm. El gas se escapa a razón de 20ml por minuto, medidas a CNPT. ¿Calcular la presión del gas que queda en el cilindro al cabo de 10 horas y la masa molar de la mezcla, asumiendo que la temperatura permanece constante a 27ºC?

PROBLEMA 28: Un frasco de 22L de capacidad contiene 40,0g de gas argón, y una masa de gas hidrógeno, medidos a una determinada presión y temperatura. La densidad de la mezcla gaseosa es de 2,00g/L. La masa atómica del argón es 40,0u. ¿Calcular:

AMADO DE DIOS ([email protected])[Escriba texto] Página 39

Autor: Amado Crisógono Castro Chonta

2015-2

a) Los gramos de hidrógeno presentes en el frasco? Rpta 40g de hidrógeno. b) La masa molar promedio de la mezcla gaseosa? Rpta. 14,7 g/mol

PROBLEMA 29: A 220ºC y 474 mmHg, una masa de 1,388g de cierta sustancia orgánica, ocupa un volumen de 420mL. El análisis del gas da una composición elemental en masa siguiente: C: 70,60%; H 5,88% y O…el resto. ¿Calcular la masa molar de la sustancia y su fórmula molecular? PROBLEMA 30: El óxido de plata se descompone completamente a temperaturas superiores a 600K, transformándose en plata sólida y oxígeno gaseoso: 2Ag2O(s)  4Ag(s) + O2(g) Cierta masa de óxido de plata da, al descomponerse 83,8 mL de oxígeno recogido sobre agua a 25ºC y 756 mmHg. La presión de vapor del agua a 25ºC es 23,8 mmHg. ¿Calcular la masa de oxígeno y de óxido de plata en la muestra?

PROBLEMA 31: Cierta cantidad de carbonato de calcio se descompone completamente por calentamiento produciendo CaO(s) y desprendiendo 2,5 L de gas CO2 que se recoge en agua medidos a 40ºC y 755,3 mmHg. La presión de vapor del agua a 40ºC es 55,3mmHg.

2, 1, 5 DETERMINACION DE MASAS MOLECULARES DE LAS

SUSTANCIAS DE BAJA TEMPERATURA DE VAPORIZACIÓN:

Diagrama Nº 10: El agua se transforma en vapor a 100ºC y 1 atm de presión

AMADO DE DIOS ([email protected])[Escriba texto] Página 40

Autor: Amado Crisógono Castro Chonta 

2015-2

Existen diferentes métodos para determinar las masas moleculares de los vapores generalmente de sustancias sólidos o líquidos a temperaturas ordinarias.





Los más conocidos son: -

Método de J.B.A. DUMAS (1826)

-

Método de VICTOR MEYER (1878)

Método de J.B.A. DUMAS: En un matraz de vidrio, de masa conocida, de unos 250 mL de capacidad volumétrica, que presenta un tubo estrecho de salida que se puede cerrar a la lámpara o con una llave, se introduce la muestra problema. Este aparato se calienta en un baño de temperatura constante, a unos 20ºC por encima del punto de ebullición de la sustancia problema. Se continúa calentando hasta que todo el líquido se ha convertido en vapor y se ha expulsado completamente el aire del matraz. El tubo estrecho del matraz se cierra luego a la lámpara o con la llave, se saca del baño caliente, se deja enfriar y se pesa. Como se conoce la masa del matraz se puede calcular la masa del vapor que lo llena, efectuando una corrección para el aire contenido en el matraz vacío. Como se conocen todos los parámetros: el volumen del matraz, la temperatura del baño y la presión atmosférica se puede calcular la masa molecular de la sustancia problema. El Método de Dumas se puede utilizar para determinar la masa molecular de sustancias que tiene temperaturas altas de vaporización, y se emplean globos de porcelana y platino.



Método de VICTOR MEYER Consiste en colocar en una ampolla de vidrio una cantidad determinada de líquido, se le evapora a temperatura constante, originando que sea desalojado del aparato un volumen equivalente de aire, que es medida en una bureta de gases, a temperatura y presión conocidas. El volumen del vapor se mide indirectamente que es igual al volumen de aire desalojado por aquel.

PROBLEMA 32: Por el método de Víctor Meyer se determinó la masa molecular de una sustancia orgánica desconocida. Después de la evaporación de 0,152 5g de la muestra líquida, se midió la expulsión de 35,05mL de aire de la bureta de gases; la presión barométrica fue de 730mmHg y la temperatura de 20ºC. El análisis químico dio la composición en masa de la sustancia: C: 22,10%;

H: 4.58%; Br: 73.32%.

Calcular: a) la masa molecular de la sustancia: b) y averiguar su fórmula molecular.

AMADO DE DIOS ([email protected])[Escriba texto] Página 41

Autor: Amado Crisógono Castro Chonta

2015-2

Solución del problema: 1º Los datos del problema son: Vmuestra = Vaire desplazado = 35,05mL = 0,035 05L P = 730mmHg

T = (20 + 273,15)K = 293,15K

m = 0,152 5g

2º Calcular la masa molecular de la sustancia: se aplica la ecuación (21,1)  L.atm  (0,1525 g )  0, 08205  (293,15 K ) mo lg.K mRT  M  ...(21,1)  M  VP  1atm  (0, 03505L)(730mmHg )    760mmHg M = 109 g/molg; entonces Rpta. (a): La masa molecular de la sustancia es 109g/molg 3º Determinar la fórmula empírica y su masa molecular de la sustancia problema: Elemento C

%m 22,10

mi(g) 24,09

H

4,58

5,00

Mi(g/molg) 12,0 1,0

ni 2 molg

 atm 2

5 molg

5

Br



73,32 79,9 79,9 1 molg 1 100,00 M =109.00g Tomando como base 109g de sustancia problema, para determinar la masa de carbono se multiplica la masa molar de la sustancia por el % en masa del carbono: mc = M.%zc = 109g(0,221) entonces la mc = 24,09g.



Para determinar el número de mol de átomos se divide la masa de carbono por su masa molecular: #atm = mc / Mc = 24,09g/(12,0 g/molg) entonces #atm = 2.

Respuesta (b): La fórmula molecular de la sustancia problema será: C2 H5 Br PROBLEMA 33: Por el método de Dumas se determinó el peso molecular de una sustancia problema (líquido a la temperatura ambiente). Unos 10 mL de esta sustancia (CCl4) se colocó en un bulbo de Dumas de material de vidrio terminando en un capilar, Se calentó luego en un baño de temperatura constante (ejemplo. H 2O en ebullición) hasta que todo el líquido se vaporizo, expulsando el aire del bulbo. Se leyó la lectura del barómetro del laboratorio, cerrándose inmediatamente el capilar del bulbo. En el momento del cierre del bulbo estaba lleno de vapor a la presión barométrica que es 1 atm a la temperatura del baño de 100ºC. Entonces “T” y “P” son conocidos, y la masa de la sustancia problema (m) se calcula por diferencia de las pesadas del bulbo cerrado y del bulbo vacío, sin vapor aunque con aire; el volumen “V” se determina llenando el

AMADO DE DIOS ([email protected])[Escriba texto] Página 42

Autor: Amado Crisógono Castro Chonta

2015-2

bulbo con agua y pesándolo. En una experiencia de laboratorio se determinaron los siguientes datos: ma = bulbo + agua

= 411g

mb = bulbo + aire

= 51,43g

mc = bulbo + CC14 (vapor) = 52,86g Solución del problema: 1º Los datos del problema son: ma = 411g

mb = 51,43 g

mc = 52,86g

P = 1atm

T = (100 + 273,15)K = 373,15K 2º La capacidad volumétrica del bulbo es V= (m a-mb)/H2O = (411 – 51)g (1mL/g)= 360mL. Entonces el volumen del bulbo es 360 mL. 3º Calcular la masa de aire en el estado inicial, en el momento de pesar el bulbo (mc): la masa de aire a las condiciones de laboratorio es 1 atm y 18ºC, se usa la ecuación (21,2)

m aire  m

aire



PVM …..(21,2) reemplazando valores RT (1atm)(0,36 L)(28,96 g / mo lg)  L.atm   0, 08205  (291,15º K ) mo lg.K 

maire = 0,44 g de aire 4º Calcular la masa del bulbo vacío es mbv = mb – maire = (51,43 – 0,44)g = 50,99g 5º Para determinar la masa de la muestra será: mCC14vapor = mc – mbv = (52,86-50,99)g = 1,87g entonces la masa es: mCC14(vapor) = 1,87g. 6º Calcular la masa molar de la sustancia: se aplica la ecuación (21,1)  L.atm  (1,87 g )  0, 08205  (373,15 K ) mo lg.K mRT  M  ...(21,1)  M  VP (0,360 L)(1atm) M = 159 g/molg; entonces Rpta. (a): La masa molar del CCl4 es 159g/molg El valor real es 154g/molg.

2, 1, 6: LEY DE GRAHAM DE LA DIFUSIÓN

AMADO DE DIOS ([email protected])[Escriba texto] Página 43

Autor: Amado Crisógono Castro Chonta

2015-2

La difusión es el proceso por el cual una sustancia se distribuye uniformemente en el espacio que la encierra o en el medio aprovechable para ella. Ejemplo1: Si se conecta dos tanques conteniendo el mismo gas a diferentes presiones, en corto tiempo la presión es igual en ambos tanques. Ejemplo2: Si se introduce una pequeña cantidad de gas en un extremo de un tanque cerrado que contiene otro gas B, rápidamente el gas A se distribuya uniformemente por todo el tanque. La difusión es una consecuencia del movimiento continúo y elástico de las moléculas gaseosas. El producto de la presión (P) de un GI por su volumen (V) es igual a los dos tercios de la energía cinética (Ec) o fuerza viva de sus moléculas PVm  Si se toma como base una molg de GI: E c 

1 Mv 2  2

Entonces la velocidad cuadrática media de un GI es:  2 

2 Ec  RT …….(48) 3

1 PVm  Mv 2  RT …..(49) 3 3RT M

3

PVm …....(50) M

Luego anunciamos los postulados siguientes:

AMADO DE DIOS ([email protected])[Escriba texto] Página 44

Autor: Amado Crisógono Castro Chonta 1.

2015-2

Las velocidades de las moléculas de un gas aumenta igualmente aumento de temperatura  

2.

3RT  M

con el

3PV …..…..(50,1) M

La energía cinética de las moléculas de un gas ideal es directamente proporcional a la temperatura absoluta: E c 

3 RT ….(51) 2

3. Y la energía cinética (Ec) promedio de las moléculas de todos los gases son idénticas a la misma temperatura constante, ecuación (51); luego la velocidad cuadrática media de las moléculas de un GI es: v2 

3P 3P 3RT 3PV    M entonces v  ……(52) p M M    V

La velocidad de difusión (v) es directamente proporcional a la velocidad de las moléculas de dicho gas (v), de acuerdo con la ley de GRAHAM (1829) v = Kv

 K  PT



1



V

v

………..(53)

A las mismas condiciones de P y T de dos gases diferentes: A y B; se puede calcular la relación entre sus velocidades de difusión: va, vb, va y vb

Para el gas A: va = Kva = K

3RT Ma

y para el gas B: vb= Kvb  K

3RT Mb

Dividiendo tenemos:

  Va Vb

K  PT 1

K

3RT Ma 3RT Mb

Para el gas A: va = Kva  K



  Va Vb

 PT 1

M b tb  …..(53,1) M a ta

3P 3P ; y para el gas B: vb = KVb  K …(53,2) a b

AMADO DE DIOS ([email protected])[Escriba texto] Página 45

Autor: Amado Crisógono Castro Chonta

Tenemos:

  Va Vb

K  PT 1

K

3P a 3P b



2015-2

V

( Va  b

b )P,T…….(53,3) a

La velocidad de difusión de un gas es inversamente proporcional a la raíz cuadrada de su respectiva masa molar o de su densidad cuando la presión del gas y su temperatura se mantienen constantes. Cuando la presión del gas y su temperatura es el mismo la velocidad de difusión del gas es inversamente proporcional al tiempo para una misma longitud de recorrido. Ejemplo cuanto más pesada es la molécula mayor es el tiempo de recorrido para una longitud dada debido a que su velocidad de difusión es menor. La Efusión es la velocidad de escape a través de un pequeño orificio, de las moléculas de un gas contenidos en un recipiente.

PROBLEMA 30: A condiciones normales de presión y temperatura (CNPT) calcular la velocidad cuadrática media de los siguientes gases: a) hidrógeno, b) nitrógeno, c) oxígeno, d) amoniaco, e) cloruro de hidrógeno, f) cloroformo, g) y metano.

Solución a: 1º Datos del problema: Volumen molar = 22,414L/molg

Masa molecular del hidrógeno = 2,016g/molg

T = 273,15K

P = 1atm = 1,013x 106dina/cm2

R = 8,314x107erg/(molg.K) 2º Calcular la velocidad cuadrática media del hidrógeno, aplicar la ecuación (52)  

3PV ……………(52) M

 

3(1, 013 x106dina / cm2)(22, 414 x103cm3) [(1g .cm / s 2) / 1dina ] (2, 016 g / mo lg)

reemplazando valores

183 814cm = 1838 m Respuesta: La velocidad cuadrática media del hidrógeno es 1838 m PROBLEMA 31: Calcular la rapidez de efusión relativa del: a) He y del O2; b) H2 y He; c) H2 y O2; y d) He y Ne.

AMADO DE DIOS ([email protected])[Escriba texto] Página 46

Autor: Amado Crisógono Castro Chonta

2015-2

Solución a: 1º Datos del problema: La masa molar del O2 y del He son, MO2 = 32,00g/nolg y MHe = 4,00g/molg. 2º Calcular la rapidez de efusión relativa del He y del O2: Asumir que los dos gases están a las mismas condiciones de presión y de temperatura y luego aplicar la ecuación

 

VHe VO 2 PT 1



  VHe VO 2

 PT 1

32, 00 g / mo lg 4, 00 g / mo lg

MO 2 ………….(53,1); MHe

Entonces

  Va Vb

PT 1

 2,83.

reemplazando

valores

Respuesta:

La rapidez de efusión relativa del He y del O2 es de 2,83

PROBLEMA 32: Calcular la temperatura a la cual la velocidad cuadrática media de la molécula nitógeno es de: a) 1000m/s, b) 800m/s, c) 2000m/s, d) y 4000m/s.

PROBLEMA 33: Un volumen de nitrógeno pasa en 24s por el orificio de un efusiómetro y bajo las mismas condiciones de temperatura y presión un volumen identico de una mezcla gaseosa de oxígeno y dióxido de carbono se difunden en 30s. ¿Calcular la composición volumétrica del CO2?

PROBLEMA 34: ¿Calcular la velocidad relativa de difusión de los gases del UF6 preparados a partir de los isótopos del uranio: U-235 y U-238. Sugerencias: usar 5 cifras significativas en los calculos?

PROBLEMA 35: El tiempo de escape de un gas diatómico a través de un pequeño orificio es de 24,4 minutos y el correspondiente tiempo del hidrógeno es de 5,5 min. ¿Cuál es la masa molar del gas diatómico?

PROBLEMA 36: Una mezcla en volumen de 6 partes de neón y una parte de argón es difundida a través de un pequeño orificio hacia un espacio evacuado. ¿Cuál es la composición de la mezcla que pasa primero?

II) GASES REALES

AMADO DE DIOS ([email protected])[Escriba texto] Página 47

Autor: Amado Crisógono Castro Chonta

2015-2

Muchos gases se comportan como gases ideales a temperaturas y presiones ambientales, es decir a presiones bajas y temperaturas altas. Si embargo la mayoría de los gases sujetos a condiciones de alta presión, se apartan de la ley de los gases ideales; y ésta proporciona una descripción cada vez más pobre de la conducta del gas. La necesidad de describir y expresar matemáticamente el comportamiento de un gas, y que concuerden con los datos experimentales nos permite señalar que en esencia, existen cuatro métodos que pueden aplicarse en los cálculos para pronosticar el comportamiento de los gases reales; ellos son: A) Las ecuaciones de estado B) Las gráficas del factor de compresibilidad C) Las propiedades calculadas D) Los datos experimentales reales Las leyes de los gases no pueden aplicarse al estado líquido. 2,2 LAS ECUACIONES DE ESTADO:

Relacionan las propiedades de P.V.T. de una sustancia pura (o de mezclas) mediante correlaciones teóricas o empíricas. Existen más de un centenar de ecuaciones de estado, de tal manera que éstas deben predecir los estados termodinámicos experimentales con exactitud razonable en toda la fase gaseosa. A continuación se indican algunas de las ecuaciones de estado, para una mol de gas: a a) Van de Waals: ( P  2 ) (V - b) = RT V Donde V es volumen molar, P es presente, R es constante universal de los gases ideales, T es temperatura, a , b y c,  y  son constante, cuyos valores dependen de las propiedades de los gases que se estudia. b)

Clausius: P =

a RT 2 V  b T (V  c )

c)

Lorentez: P =

RT a 2 (V +b) V V2

AMADO DE DIOS ([email protected])[Escriba texto] Página 48

Autor: Amado Crisógono Castro Chonta

d)

Berthelot: P =

e)

Keyes: P =

donde: f)

2015-2

a RT V  b TV 2

A RT ; (V  1) 2 V S

S = e - /V

Ecuación virial de estado.- Es una ecuación de la forma: B (T ) C (T ) PV =1+ + + ……. RT V V2

2,2,1 Ecuación de estado de Van der Waals VDW): Es la primera ecuación de estado propuesto, por Van der Waals, y se uso, para describir el comportamiento de un gas real. Es a la vez la ecuación de estado más simple. Van Der Waals introdujo en la ley del gas ideal los siguientes conceptos:  El efecto de las fuerzas de atracción existente entre las moléculas, es decir cuando una molécula alcanza la pared del recipiente que lo contiene, choca con menor intensidad, debido a que las moléculas adyacentes ejercen fuerzas de atracción, lo que es lo mismo disminuye la presión ejercida contra la pared o presión del gas. Entonces la presión corregida del gas será: (P +

a ) V2

Donde (a) es una constante dimensional que depende de la naturaleza de las fuerzas atractivas del gas.  El efecto de volumen, se debe a que las moléculas de un gas real tiene un volumen libre disponible o volumen compresible que es la diferencia entre el volumen total molar (V) y el volumen propio de las moléculas (b). La constante (b) de VDW es en realidad cuatro veces el volumen real de las

AMADO DE DIOS ([email protected])[Escriba texto] Página 49

Autor: Amado Crisógono Castro Chonta

2015-2

moléculas, que esta determinada por la mínima distancia que dos moléculas pueden acercarse. Entonces el volumen corregido del gas será: (V - b) La ecuación de Van de Waals será: Para una mol

(P +

a ) (V - b) = RT V2

……. (1)

Para n moles

(P +

 2a ) (V - b) = RT V2

……. (2)

Las constantes a y b se determinan aplicando la ecuación de VDW a los datos experimentales P, V, T y particularmente a los valores en el punto crítico, entonces la ecuación de VDW es una expresión semiempírica. En la Tabla # 1 se puede encontrar los valores de las constantes a y b de VDW de algunos gases. En caso de que no se encontraran, se tiene que calcular teniendo en cuenta que la isoterma crítica pasa por el punto de inflexión en el punto crítico y que la pendiente vale cero en ese punto, (@P / @V) T = 0, igualmente la segunda derivada, (@2p / @V2) = 0, por lo tanto para la ecuación de estado de VDW (1), ordenando tenemos: P=

a RT - 2 V b V

……. (3)

Tomando la primera y segunda derivada a la ecuación (54a) tenemos:  P     V 

 2R    2   V 

=T

=T

RT 2a 2 + (V  b) V3

2 RT 6a 3 + (V  b) V4

……………. (4)

……. (5)

Debido a que las dos derivadas son iguales a cero en el punto crítico (Tc, Pc, Vc) tenemos: -

RTc RTc 2a a = 0  = 2 + (Vc  b) 2(Vc  b) 2 Vc 3 Vc 3

……. (6)

-

2 RTc RTcVc 6a a + 4 = 0  3 = (Vc  b) 3 3(Vc  b)3 Vc Vc

……. (7)

AMADO DE DIOS ([email protected])[Escriba texto] Página 50

Autor: Amado Crisógono Castro Chonta Pc =

2015-2

RTc a (Vc  b) Vc 2

…..… (8)

Los primeros miembros de las ecuaciones (6) y (7) son iguales, luego: RTc RTcVc  Simplificando se obtiene que: 2 = 2(Vc  b) 3(Vc  b) 3

V c = 3b

……. (9)

Y al reemplazar en (6) y simplificando se obtiene que: a = (9/8) RTc Vc

……. (10)

Luego reemplazando (9) y (10) en (8), tenemos: Pc = (3/8) RTc/ Vc

……. (11)

Combinando las ecuaciones (8), (9), (10) y (11) se demuestran que: Pc = a/ (27b2)

……………. (12)

8a ( 27 Rb)

……………. (13)

a=

27 R 2Tc 2 64 Pc

……………. (14)

b=

RTc 8 Pc

Tc =

……. (15)

Las unidades de las constantes a y b son: a < ---- > atm (cm3/molg)2

;

psia (pie3/mol/b)2

Pa (m3/molg)2 b < ---- > cm3/molg;

pie3/mol lb;

m3/molg

La ecuación de VDW puede resolverse fácilmente para la presión (o para la temperatura): Para una mol : P =

a RT - 2 V b V

RT  2a Para n moles: P = V  b V2

……. (3)

o

……. (3a)

En caso de resolver el volumen (v) o moles (n) la ecuación (2a) se convierte en una relación de tipo cúbico:

AMADO DE DIOS ([email protected])[Escriba texto] Página 51

Autor: Amado Crisógono Castro Chonta V3 - (b +

RT P

)V2+(

 2a P

)V-

2015-2

 3 ab =0 P

……. (2b)

En consecuencia para determinar el volumen o moles se tendría que: a) Utilizar el método de aproximaciones sucesivas. b) Obtener la gráfica de L ecuación suponiendo diferentes valores de V (o de n), y observar a que valor de V (o de n) en la curva se logra la intersección con la abscisa. c) Emplear el método de NEWTON.

TAB LA 1 Constantes a y b de la ecuación de VDW y constantes críticas de algunos gases:

CONSTANTES a Y b DE LA ECUACIÓN DE VDW Y CONSTANTES CRÍTICAS DE ALGUNOS GASES: GAS

M

a atm

b 3

(cm /molg)

Tc ºK

Pc atm

Vc cm /molg 3

AMADO DE DIOS ([email protected])[Escriba texto] Página 52

Autor: Amado Crisógono Castro Chonta

Aire Ar Cl2 CH4 C2H2 C2H4 C2H6 C3H8 C4H10 C6H6 C6H5CH3 C6H5Cl C6H5Br CH3OH C2H5OH CH3Cl CHCl3 CCl4 CO CO2 CS2 CH3COOH (C2H5)2O He H2 HCl HBr H2O(g) Ne N2 NO NO2 N2O NH3 O2 SO2

28,970 39,948 70,910 16,040 26,040 28,050 30,070 44,098 58,120 78,110 92,142 112,560 157,010 32,040 46,070 50,490 119,390 153,840 28,010 44,010 76,140 60,050 74,120 4,000 2,016 36,470 80,917 18,016 20,183 28,020 30,006 46,010 44,020 17,034 32,000 64,063

(L/molg)2 1,328 1,345 6,495 2,255 4,368 4,476 5,490 8,668 14,312 18,000 24,062 24,433 28,581 9,525 12,023 7,471 15,170 20,380 1,485 3,592 11,630 17,630 17,400 0,034 0,244 3,668 4,453 5,478 0,211 1,347 1,340 5,295 3,782 4,187 1,362 6,718

2015-2

36,62 32,20 56,22 42,78 51,23 57,16 63,80 84,42 121,15 115,20 146,35 145,33 153,93 67,05 84,08 64,86 10,22 138,34 39,83 42,78 76,88 10,72 13,44 23,72 26,62 40,82 44,22 30,63 17,08 38,62 27,96 45,38 44,15 37,32 31,88 56,52

132,6 151,2 417,2 190,7 309,5 283,1 305,4 369,9 425,2 562,6 594,2 632,2 670,2 513,4 516,2 416,5 536,2 556,4 133,2 304,2 552,0 594,2 467,4 5,3 33,3 324,6 363,3 647,4 44,5 126,2 179,2 432,0 309,8 405,5 154,4 430,7

37,20 48,00 76,10 45,80 61,60 50,50 48,20 42,00 37,50 48,60 41,60 44,60 44,60 78,50 62,99 65,80 54,20 45,00 34,56 72,88 78,30 57,35 35,60 2,26 12,80 81,53 84,00 218,50 25,90 33,50 64,80 100,00 71,80 111,30 49,70 77,80

92,59 74,94 124,00 98,78 113,12 124,32 147,96 200,98 254,96 260,05 315,53 308,46 323,22 117,78 167,32 143,43 238,37 276,20 93,20 94,23 172,70 172,30 281,60 58,30 65,40 87,30 55,60 41,70 90,20 58,10 82,30 96,80 72,50 74,20 122,20

Equivalencias de unidades para pasar de un sistema a otro sistema de unidades: 1,000 00 atm (L/molg)2 = 256,591 atm (pie3/mollb)2 = 106 atm (cm3/molg)2= 3,771 x 103 psia (pie3/mollb)2 1,000 dm3/molg 1lb = 453,592 37g;

= 103 cm3/mol

= 16,018 pie3/mollb

1kg = 1000g = 2,205 lb;

1TM = 1000kg = 2 205lb.

AMADO DE DIOS ([email protected])[Escriba texto] Página 53

Autor: Amado Crisógono Castro Chonta

2015-2

PROBLEMA 1: Calcular la presión ejercida por un mol de CO2 que ocupa el volumen de 10,0 L a 100ºC por dos métodos: a)

La ecuación de los gases ideales y

b)

La ecuación de VDW

SOLUCIÓN: 1. Base de cálculos y datos del problema V = 10,0 dm3/molg

T = 373,2ºK

 = 1,0 molg

R = 0,082 057 dm3.atm/molg K

a = 3,60 atm (dm3/molg)2;

b = 0,0428 dm3/molg (datos de la tabla 1)

2. Por el método de gases ideales la presión será: P = RT/V  P = 0,082 057 (L atm/molg K)(373,2K)/(10,0L/molg) P = 3,06 atm: Rpta (a) 3. Por el método de VDW, la presión será: P=

a RT - 2 V b V

(0, 082 057 L atm / mol K )(373, 2 K ) P= (10, 0 L / mo lg  0, 0428 L / mo lg)

3,60 atm( L / mo lg) 2 (10,0 L / mo lg) 2

P = 3,04atm: Rpta (b) 4. Discusión y Conclusión: Se observa que el gas, CO 2, a las condiciones dadas se comporta como un gas ideal porque el resultado obtenido por los dos métodos es prácticamente igual. PROBLEMA 2: Calcular por dos métodos el volumen ocupado por una mol de O 2 a -88ºC y 44,7 atm 1. Base de cálculo y datos del problema  = 1,00 molg O2 ;

T = 185,2K

P = 44,7 atm

;

a = 1,36 atm (L/molg)2

b = 0,031 9 L/molg 2. Por el método de gases ideales: V = RT/P, reemplazando valores tenemos:

AMADO DE DIOS ([email protected])[Escriba texto] Página 54

Autor: Amado Crisógono Castro Chonta

2015-2

V = (0,082 057 L atm/molg K) (185,2K/44,7 atm) V = 0,34 L/molg: Respuesta 3. Aplicando la ecuación de VDW: (P +

a ) (V – b) – RT = 0 V

 Resolviendo por el método de aproximaciones sucesivas y gráfico 1ro. Reemplazar los valores en la ecuación (1) (44,7 +

1,36 ) (V – 0,031 9) – 0,082 06 (185,2) = 0 V

2do. Hacer una tabla, dar valores a V y calcular  1,36 ) (V – 0,0319) – 15,197 5 V2



V

(44,7 +

0,34

(56,4647) (0,3081) – 15,197 5

2,201

0,25

(66,46) (0,2181) – 15,197 5

-0,701

El valor que se busca (V) esta entre 0,34 cercano a 0,25 L/molg; para encontrarlo graficar  vs V

0,27

1.0

Respuesta: En este caso el valor será 0,27 1/molg que corresponde al intersepto con la abscisa. 2,2,2 FACTORES DE COMPRESIBILIDAD Se discutirá la ecuación de estado del factor de compresibildiad (Z) y los métodos para su cálculo. En los primeros experimentos se encontró que en el punto crítico todas las sustancias están aproximadamente en el mismo estado de dispersión

AMADO DE DIOS ([email protected])[Escriba texto] Página 55

Autor: Amado Crisógono Castro Chonta molecular,

entonces

será

conveniente

2015-2 considerar

que

las

propiedades

termodinámicas y físicas deberían ser similares. Estas observaciones originó la ley de los estados correspondientes, que expresa que en el estado crítico todas las sustancias deben comportarse en forma semejante. El estado crítico (Pc, Tc, Vc), es el conjunto de condiciones físicas en las cuales la densidad y otras propiedades del líquido y del vapor son idénticas. En particular solo para el caso de un componente puro, es la máxima temperatura a la cual puede existir en equilibrio el líquido-vapor. Los valores experimentales de la Temperatura crítica (Tc) y Presión crítica (Pc) para sustancias puras pueden obtenerse de la Tabla 1 en caso que no se encuentre consultar los textos de Reid y Sherwood (5), capítulo 2, y el manual del Ingeniero Químico (6), pags. 3-142, 3-143, 3-287, 3-288, 3-289; en las cuales se describe y se analizan los métodos para determinar las constantes críticas de diversos compuestos y elementos.

AMADO DE DIOS ([email protected])[Escriba texto] Página 56

Autor: Amado Crisógono Castro Chonta

2015-2

Al correlacionar las propiedades de estado (P, V, T) de los gases con su estado crítico (Pc, Tc, Vc), se observó que todas las sustancias se comportan en forma similar en su estado reducido (Pr, Tr, Vr); particularmente cualquier sustancia tiene el mismo volumen reducido (Vr) a la misma temperatura (Tr) y presión reducida (Pr) ver Figura 1. Las propiedades reducidas se calculan de la siguiente manera: Vri =

V …… (16); Vci

Tr =

T …… (17) Tc

y

Pr =

P …… (18) Pc

Matemáticamente se puede expresar que la ecuación funcional en el estado reducido es f(Pr,Tr,Vr)=0 …………… (19) FIGURA 1: Comprobación Experimental de la relación PR /TR = cte/ VR para un conjunto de gases (metano, etano, propano, pentano, heptano y etileno). VR=0.80 VR=1.00

VR=0.60

PR

VR=1.50

VR=2.00

3.00

4.00

AMADO DE DIOS ([email protected])[Escriba texto] Página 57

Autor: Amado Crisógono Castro Chonta

2015-2

TR

2,2,2a ECUACIÓN DE ESTADO DEL FACTOR DECOMPRESIBILIDAD Se expresa como:

PV = ZRT PV= ZRT

…………… (20)

o

…… (21)

Es una modificación de la ecuación de estado de un gas ideal, que describe el comportamiento (P, V, T) de los gases reales a lo largo de una variedad mucho más amplia de condiciones. Ha sido desarrollada para relacionar los conceptos de la ley de los estados correspondientes y la ley del gas ideal. La cantidad adimensional (Z) se denomina factor de compresibilidad. Un valor de Z = 1.000 corresponde a un comportamiento de un gas ideal. ESTIMACIÓN El factor de compresibilidad de un gas depende de la temperatura y de la presión del gas, Z = f(T, P). Z varía de un gas a otro para un mismo valor de T y P. ejemplo: el valor de Z para N2 a 200ºK y 100 bar es 0,845 5, mientras que el valor de Z para el CH 4 a las mismas condiciones de P y T es de 0,365 7

Fig. 2 Factor de compresibilidad como función de la temperatura y la presión

Fig. 3 Compresibilidad como una función de la temperatura y presión reducidas

AMADO DE DIOS ([email protected])[Escriba texto] Página 58

Autor: Amado Crisógono Castro Chonta

2015-2

En la Biblioteca del Ingeniero Químico (6) pags 3-143 a 3-147, incluyen valores de Z = f(T, P) para el aire, Argón, CO2, CO, H2, CH4, N2, O2, y Vapor de agua PROBLEMA 3: Se almacenan 388 kg de H2 en un tanque a 150ºK y 70 atm de presión absoluta. Estimar el volumen del tanque utilizando la ecuación de estado del factor de compresibilidad. SOLUCIÓN: a. Base de cálculos: 388 kg de H2 b. El número de molkg de H2 será:

nH2 = 388 kg

 1molkg H 2     2,016 kg 

388

nH2 = 2,016 molkg H2 c. El valor de Z para H2 será 1,050 7 obtenido de la bibliografia (6) (pág 3145):

T = 150ºK y P = 70 atm  Z = 1,0507

d. Usar la ecuación PV = ZRT para calcular el volumen  V = ZnRT/P, reemplazando valores tenemos: 

388  molkg  V = 1,0507   2,016 

 150 K     70 atm

 0, 082057m3 atm   K molkg  

V = 35,56 m3 Sin embargo al construir las gráficas del factor de compresibilidad con respecto a los estados reducidos Pr y Tr, Z = z (Tr, Pr), se observa que para todos los gases los valores de Z a la misma Tr y Pr coinciden aproximadamente en un mismo punto, ver figura Nº 3. Entonces para estimar Z=z (Tr, Pr) se usarán las Gráficas 4 al 8 del factor de

AMADO DE DIOS ([email protected])[Escriba texto] Página 59

Autor: Amado Crisógono Castro Chonta

2015-2

compresibilidad generalizado, que fueron preparados por Nelson y Obert (7). El procedimiento a seguir para estimar Z es el siguiente: 1. Buscar o estimar valores de Tc y Pc 2. Si el gas es H2 ó He, corregir los valores críticos: (Tc) corregido = Tc + 8 ºK

…… (22)

(Pc) corregido = Pc + 8 atm

…… (23)

3. Calcular los valores reducidos de las variables conocidas, mediante las fórmulas: Tr = T/Tc

…… (17)

Pr = P/Pc

…… (18)

Vri = V/Vci

…… (24)

Vci = RTc/Pc

…… (25)

Donde: Vri es volumen molar reducido ideal y Vci es volumen molar crítico ideal. Las constantes reducidas deben ser adimensionales. 4. Una vez calculadas dos de las propiedades reducidas usar la gráfica del factor de compresibilidad, que ha sido seleccionado para estimar Z. PROBLEMA 4: Un cilindro de 5,00 pie3 contiene 50,00 lb de propano (C3H8) y se encuentra en un lugar, donde recibe directamente el calor de un caldero. El manómetro indica que la presión es de 665 psig. ¿Cuál es la temperatura del C 3H8 en el cilindro, calcular por 3 métodos? a) Gases ideales b) Ecuación de estado de VDW y c) Ecuación de estado del factor de compresibilidad (Z). SOLUCIÓN: 1. Base de cálculos: 50,00 lb de C3H8 2. Por el métodos de gases ideales, la temperatura del gas será: T = PV/nR

reemplazando valores tenemos:

AMADO DE DIOS ([email protected])[Escriba texto] Página 60

Autor: Amado Crisógono Castro Chonta (679, 7 psia) (5, 00 pies 3 ) psia pie3 ( T= (50, 0 lb) (10, 73 ) mol lb R

2015-2

44,098 lb ) 1 mol lb

T = 279,3R: Rpta (a) 3. Por el método de gases reales, ecuación de estado de VDW Ecuación T = (P +

 2 a V  b ) ( R ) reemplazando valores tenemos: V2

n = 50,0 lb (1molb/44,098 lb) entonces

T = (679,7 +

50,00  3,27 x10 4    44,098  (5,0) 2

 50,000  1,35   44,098  )  50,000  10,73    44,098 

5,0  

2



n = (50,0/44,098) mollb

)(

T = 673,3R: Rpta (b) 4. Por el método del factor comprensibilidad, Z: a) Construir la siguiente tabla: Pc (atm)

Tc (ºK)

Vci (L/molg)

Pr

Vri

42,0

369,9

0,7226

1,10

0,381

V (L) 0,275

Z

0,405

b) Calcular Vci = RTc/Pc Vci =

(0, 082057

L atm ) (369,9 K ) mol K , luego Vci = 0,722 7 L/molg 42, 0 atm

c) Calcular Vri = V/Vci

reemplazando valores tenemos:

28,31685 L 2, 205 mol lb 5, 0 pie3 ) ( ) ( ) 3 1, 00 pie 103 mo lg Vri = 50, 00 / 44, 098) mollb 0, 7227 L / mo lg (

Vri = 0,381 d) Calcular Pr = P/Pc, reemplazando valores tenemos:

AMADO DE DIOS ([email protected])[Escriba texto] Página 61

Autor: Amado Crisógono Castro Chonta

Pr =

2015-2

 679,7 psia   1,00 atm       42,0 atm   14,696 psia 

Pr = 1,10 e) Calcular Z usando el diagrama de Z: f) Calcular T =



Z = 0,405

1 (PV/nR) Z

Reemplazando valores tenemos: T = 279,3 R/0,405 T = 689,6 R: Rpta (c) La respuesta más confiable es T= 689,6R por que se calculó por el método del factor de comprensibilidad (Z) PROBLEMA 5: Un reactor de amoniaco gaseoso se llenó con este reactivo equivalente a 8,0 lb. La presión en el reactor es de 150 lbf/pulg 2 abs y la temperatura es de 250ºF. Determinar el volumen del reactor utilizando tres métodos: a) Gases ideales b) Ecuación de VDW c) Factor de compresibilidad SOLUCIÓN: 1. Base de cálculos: 8,00 lb de NH3 P = 150 psia

T = 709,67 R

M = 17,034 lb/mollb

n= (8,00/17,034) mollb  n = 0,469 65 mollb  R = 10,73 (psia pie3/mollbR) 2. Por el método de gases ideales, el volumen del gas será: V = nRT/P V=

(0, 469 65 mol lb) (10, 73 psia. pie3 / mollbR) 709, 67 R 150 psia

AMADO DE DIOS ([email protected])[Escriba texto] Página 62

Autor: Amado Crisógono Castro Chonta V = 23,84 pie3 :

2015-2

Rpta (a)

3. Por el método de gases reales, utilizando la ecuación de estado de VDW. (P +

 2a ) (V - b) – RT = 0 V2

…… (2)

a = 15 800 psia (pie3/mol lb)2 b = 0,597 5 pie3/mol lb Reemplazando valores en la ecuación de VDW:

15 800(0,4700) 2 (150 + V2

) (V - 0,47000(0,5975) - 0,47 (10,73) (709,67) =

0 Construir una tabla dando valores a la incognita (V) 3490,22 ) (V - 0,281) V2 Construyendo factores y sumandos:

V

(150 +

3578,9



1º 2º 3º 4º 3 V (pie ) 24,0 156,060 23,719 3578,94 122,65 23,0 156,600 22,719 3578,94 -21,2 23,2 156,485 22,919 3578,94 7,5  Construir una gráfica con los puntos calculados (V, ) para determinar el volumen, de tal manera que el V = 23,15 pie3 cuando  = 0,000 entonces el V = 23,15 pie3

Respuesta…… (b)

7.5

23,15

AMADO DE DIOS ([email protected])[Escriba texto] Página 63

Autor: Amado Crisógono Castro Chonta

2015-2

23,0 21,2

4. Por el método del factor de compresibilidad (Z) a) Construir la siguiente tabla: Tc (K)

Pc (atm)

405,5 111,3

P psia

T (R)

150

709,67

Tr

Pr

0,9723

0,091 7

Z

0,963

b) Calcular Tr = T/Tc Tr =(709,67R/405,5K) (1,0K/1,8R)

entonces

Tr = 0,972 3

c) Calcular Pr = P/Pc Pr = (150 psia/111,3 atm) (1,00 atm/14,696 psia) luego Pr = 0,091 7 d) Calcular Z usando las gráficas del factor de compresibilidad Z = 0,963 e) Calcular V = ZnRT V = 0,963 (23,84 pie3)



El volumen del reactor es 23,0 pie3

V = 22,96 pie3 Rpta (c)

La respuesta más confiable es V = 23 pie3 calculado por el método del factor de comprensibilidad (Z).

PROBLEMA 6: Calcular el valor de Z en el estado crítico usando las gráficas del factor de compresibilidad. SOLUCIÓN: 1. En el punto crítico los valores de Tr y Pr serán: Pr = Pc/Pc  Pr = 1,000 Tr = Tc/Tc  Tr = 1,000 2. Usar la gráfica # 8 para estimar ZTr y luego calcular (Z): Z Tr = 0,27  Z = 0,27

AMADO DE DIOS ([email protected])[Escriba texto] Página 64

Autor: Amado Crisógono Castro Chonta

2015-2

El valor de Zc es 0,27 PROBLEMA7: Un tanque de oxígeno contiene 100,000 pie3 de 02 seco medidos a 400K y 10 bar. Se desea pasar este gas a un tanque cuya capacidad es 10 pie 3 y que se encuentra a la temperatura de 220K ¿Cuál será la presión en el tanque; calcular usando 3 métodos? SOLUCIÓN: 1. Base de cálculos y datos del problema: Estado (1)

P

V

10 bar = 9,869 2atm

(2)

T

100 pie3 = 2831,7L

400K

10 pie3 = 283,17 L

?

220K

2. Por el método de gases ideales, la presión P2 será: P2 = 10 bar (100 pie3/10pie3) (220K/400K) P2 = 55 bar

o

P2 = 54,28 atm

Las molgO2 que se encuentran en el recipiente será: nO2 = PV/RT nO2 = (9,869 2atm) (2831,7L)/(400K)(0,082 057 L atm/(mol.K)) nO2 = 851,44 molg 3. Por el método de la ecuación de VDW Calcular el número de molg usando los datos del estado (1) 3,1 Primero hacer una tabla: Ecuación: a = 1,36 atm (1/molg)2

(P +

n2 a ) (V - nb) - nRT = 0 V2

b = 0,031 9 a/molg

n 2 (1,36) (9,8692 + ) (2831,7 - 0,031 9n) - 0,082 057 (400) n= 0 2831, 7 2 (9,8692 + 1,696 x 10-2n2)

n

(2831,7-0,031 9n) -32,822 8n

850

9,992 00

2804,6

27 899,4

860

9,994 64

2804,3

28 277,6



124 -199

n = 853,8 molg 3,2 Calcular P2 usando los datos del estado (2)

AMADO DE DIOS ([email protected])[Escriba texto] Página 65

Autor: Amado Crisógono Castro Chonta P=

P2

2015-2

nRT n2a V  nb V 2

853,8 (0,082 06) (220) 283,17  853,8 (0,031 9)

=

-

853,8 2 (1,36) 283,17 2 P2 = 47,86 atm

P2 = 48,49 bar

4. Por el método de las gráficas del factor de compresibilidad: Hacer una tabla para calcular Z y n con los datos del estado (1)

4,2

Pc

Tc

(atm)

(ºK)

49,7

154,4

Pr

Tr

0,199

2,59

Z

 = PV/ZRT

1,000

851,44 molg

Hacer una tabla para calcular Z y P2 com los datos del estado (2) Pc

Tc

Pr

(atm)

(ºK)

1/molg

49,7

154,4

0,3326

Vri

Tr

Z

P2 (bar)

1,30

1,425

0,894

49,17

a) Calcular: Vci =

RTc  Vci Pc

0,082 06(154,4) 49,7

Vci = 0,255 L/molg b) Calcular:

AMADO DE DIOS ([email protected])[Escriba texto] Página 66

Autor: Amado Crisógono Castro Chonta

V=

283,17 1 851,44 mo lg

2015-2

 V = 0,332 6 L/molg

c) Calcular:

0,332 6 L / mo lg Vri = V/Vci  Vri = 0,255 L / mo lg Vri = 1,30 d) Calcular: Tr = T/Tc  Tr = 220ºK/154,4ºK Tr = 1,425 e) Calcular Z usando el gráfico de factor de compresibilidad  Z = 0,894 f) Calcular: P2 = Z (RT/V)



P2 = 0,894 (55 bar)

P2 = 49,17 bar

o

P2 = 48,53 atm

2,3 MEZCLAS GASEOSAS DE LOS GASES REALES Muchos de los problemas prácticos en la industria y en la mayoría de los casos manejamos mezclas gaseosas, por lo tanto discutiremos diferentes métodos para calcular las propiedades (P, V, T) de mezclas en base exclusivamente a las propiedades de los componentes puros: A) Ecuación de estado B) Constantes promedios C) Factor de compresibilidad medio (Zm)

AMADO DE DIOS ([email protected])[Escriba texto] Página 67

Autor: Amado Crisógono Castro Chonta

2015-2

2,3A ECUACION DE ESTADO Cada una de las ecuaciones de estado para gases reales y puros se puede combinar con las leyes de Dalton o las leyes de Amagat. Ejemplo las leyes de Dalton y Amagat pueden combinarse con la ecuación de estado de VDW. 2,3A1) LA ECUACION DE ESTADO Y LA LEY DE DALTON Usando la ecuación de estado de un gas real se calcula la presión parcial de cada componente puro, para luego sumarles de acuerdo con la ley de Dalton y obtener la presión total del sistema. Entonces cuando se usa la ecuación de estado de VDW y la ley de Dalton tenemos las siguientes expresiones matemáticas: P = Pa + Pb + Pc + ....…… (26) La presión parcial Pj para cada componente será: 2  a RT  a - a a V   a ba V2

…… (27)

2  b RT  b ab Pb = V   b bb V2

…… (28)

Pa =

Reemplazando las ecuaciones (27) y (28) en (26) tenemos la presión de la mezcla: (Pt): Pt = RT (

 a RT  b RT 1 2 2 + + ...) 2 (a aa + b ab + ...) V   a ba V   b bb V

……

(29)

2,3A2) LA ECUACION DE ESTADO Y LA LEY DE AMAGAT Para calcular V(o ) de una mezcla; primero debe calcularse V (o ) de cada componente y la suma de estos es el volumen total según la ley de Amagat. La resolución de este problema es muy complicado. Para el caso particular de la ecuación de estado de VDW y la ley de Amagat, tenemos las siguientes expresiones matemáticas: V = Va + Vb + Vc + .....

…… (30)

AMADO DE DIOS ([email protected])[Escriba texto] Página 68

Autor: Amado Crisógono Castro Chonta

2015-2

y luego estimar los valores de Va, Vb, Vc etc. (o a, b, c, etc.) usando la ecuación de VDW. Para cada componente V (o n) es una ecuación cúbica cuyo cálculo es complicada. 2,3B CONSTANTES PROMEDIO Una forma simple y efectiva es usar las constantes promedio de la ecuación de estado de un gas real, que permitirá resolver problemas de mezclas gaseosas. Para el caso particular de la ecuación de estado de VDW se puede proceder como sigue: bm = baYa + bbYb. + ..... a1/2m= a1/2aYa + a1/2Yb + .....

…… (31) …… (32)

Las constantes promedio también pueden obtenerse a partir de las constantes seudocríticas; entonces para la ecuación de VDW tenemos:

a=

27 R 2Tc 2 64 Pc 2

…… (33)

b=

RTc 8 Pc

…… (34)

y

2,3C FACTOR DE COMPRESIBILIDAD MEDIO (Zm) Para mezclas de gases se puede usar la ecuación de estado del factor de compresibilidad: PV = Zm (RT)

…… (35)

Donde Zm es el factor de compresibilidad medio y se puede calcular usando las siguientes expresiones: Zm = ZaYa + ZbYb + .....

…… (36)

AMADO DE DIOS ([email protected])[Escriba texto] Página 69

Autor: Amado Crisógono Castro Chonta

2015-2

Sabemos que Z es una función de Pr y Tr; entonces es necesario decidir que presión deberá usarse para calcular Pr. 2,3C.1) CONSIDERANDO LA LEY DE DALTON Cada componente Zj se calcula a la Trj y a la presión parcial reducida de cada componente gaseoso Prj. Esta se expresa como: Prj = Pj/Pcj

…… (37)

Trj = T/Tcj

…… (38)

y

2,3C2 DE ACUERDO CON LA LEY DE AMAGAT Cada componente Zj se calcula a la Trj y P Rj total del sistema. Esta se expresa como: Prj = P/Pcj

…… (39)

Trj = T/Tcj

…… (40)

2,3D PROPIEDADES SEUDOCRÍTICAS Conocido como el método de Kay. Los valores seudocríticos se calculan según las expresiones siguientes: Presión Seudocrítica: P'c = PcaYa + PcbYb + .....

…… (41)

Temperatura Seudocrítica: T'c = TcaYa + PcbYb + .....

…… (42)

Temperatura Seudocrítica ideal: V'c1 = (Vci Y)a + (Vci Y)b + .....

Luego (Vci)a =

RTc a RTc b ; (Vci)b = Pc a Pcb

…… (43)

…… (44)

AMADO DE DIOS ([email protected])[Escriba texto] Página 70

Autor: Amado Crisógono Castro Chonta

2015-2

Los respectivos valores seudoreducidos son: Presión Seudoreducida  P'R = P/P'c

…… (45)

Temperatura Seudoreducida  T'r = T/T'c

…… (46)

Volumen Seudoreducida  V'r = V/V'ci

…… (47)

Las propiedades seudoreducidas se usan en la misma forma que las propiedades reducidas de una sustancia pura:

PROBLEMA # 8 Una mezcla gaseosa tiene la siguiente composición: Eteno 57,00%, Argón 40,00%, Helio 3,00%. A 120 atm de presión y 25ºC el volumen determinado experimentalmente es 0,144 L/molg. Comparar con el volumen molar calculado por los siguientes métodos: a) Gases ideales b) Presiones parciales más el Z c) Volúmenes parciales más el Z d) Método de Kay e) Valores medios de VDW f) Calcular el % de error para cada método de (a) a (e). SOLUCIÓN: 1,0 Base de cálculos: 1,000 molg 2,0 Por el método de gases ideales: 2.a) El volumen del gas será: V = RT/P

V=

(0,082 057 L atm / mo lg k ) (298,2º K ) 120 atm

V= 0,204 1/molg

Rpta (a)

AMADO DE DIOS ([email protected])[Escriba texto] Página 71

Autor: Amado Crisógono Castro Chonta

2015-2

2.b) El % de Error será: %E =

0,204  0,144 x 100 0,144



%E = 41,7% 3,0 Por el método de Presiones parciales más el Z: 3.a) Construir la siguiente tabla para calcular Zm: COMP ONENT E Eteno Argón Helio

Yj

Pj (atm)

0,5 68,40 7 0,4 48,00 0 0,0 3,60 3

Tj (ºK)

Pc (atm)

Tc (ºK)

298,2 50,50 283,1 0 0 298,2 47,94 150,6 0 5 298,2 2,26+ 5,26+ 0 8 8

Prj

1,35 4 1,00 1 0,35 1

Trj

Zj

1,053 0,38 3 1,979 0,98 0 22,49 1,00 0 5

Yj Zj 0,218 3 0,329 0 0,030 2

Zm

0,640 5

3.b) Calcular el volumen molar (L/molg) usando la ecuación: V = ZmRT/P  V = 0,640 5 (0,204 L/molg) V = 0,131 L/molg

Rpta (b)

3.c) El % de Error será: %E =

 (0,144  0,131) x 100  % E = - 9,03 % =  9,03% 0,144

4,0 Por el método de volúmenes parciales más el Z: 4.a) Construir la siguiente tabla para calcular Zm: COMP ONENT E Eteno Argón Helio

Yj

P (atm)

0,5 120,0 7 0 0,4 120,0 0 0 0,0 120,0 3 0

T (ºK)

Pc (atm)

Tc (ºK)

Prj

298,2 0 298,2 0 298,2 0

50,50

283,1 0 150,6 5 13,26

2,38 0 150, 65 13,2 6

47,94 10,26

Trj

Zj

1,053 0,38 3 1,979 0,96 7 22,49 1,04 0 0

Yj Zj 0,218 3 0,386 8 0,031 2

Zm

0,636 3

AMADO DE DIOS ([email protected])[Escriba texto] Página 72

Autor: Amado Crisógono Castro Chonta

2015-2

4.b) Calcular el volumen molar (L/molg) usando la ecuación: V = ZmRT/P  V = 0,6363 (0,204 l/molg) V = 0,130 L/molg

Rpta (c)

4.c) El % de Error será: %E =

 (0,144  0,130) x 100 0,144

%E =  -9,72% =  9.72% 5,0 Por el método de Kay: 5.a) Construir la siguiente tabla para calcular Zm: Yj COMPONE NTE Eteno Argón Helio

P´r =

P P' c

Pc (atm)

0,57 0,40 0,03

50,50 47,94 10,26

 P'r =

Tc (ºK)

Yj Pc (atm)

283,10 150,65 13,26

28,79 19,18 0,31 P'c = 48,27

Yj Tc (K)

p'r

161,40 60,30 2,49 0,40 222,10 = T'c

120 atm 48,27 atm

P´r = 2,49 T´r =

T T 'c

298,2 K

 T'r = 222,1K

T´r = 1,34 Usar la gráfica #6 para estimar Zm = 0,70 V= 0,70(0,204 L/molg) entonces V= 0,143 L/molg Rpta (d) %E =

 (0,144  0,131) x 100  % E = -0,70 % =  0,70% 0,144

6,0 Por el método de los valores medios de VDW 6.a) Construir la siguiente tabla para calcular los valores medios de bm y am Yj COMPONE NTE Eteno Argón

0,57 0,40

bj aj (L/mol (l/molg)2a g) tm 0,0572 4,48 0,0323 1,35

Yj bj 0,032 60 0,012 92

(aj)1/2 Yj 1,206 46 0,464 76

am

2,8115

AMADO DE DIOS ([email protected])[Escriba texto] Página 73

T´R

1,34

Autor: Amado Crisógono Castro Chonta Helio

0,03

0,0237

2015-2 0,034 0,000 71 bm = 0,046 23

0,005 53 1,676 75 = am1/2

6.b) Calcular el V por el método de aproximaciones sucesivas. La ecuación de estado de VDW para mezclas es: (P +

am ) (V - bm) - RT = 0 V2

(120 +

2,8115 ) (V - 0,04623) - 0,082057) (298,2) = 0 V2

Luego elaborar una tabla dando valores a V para calcular  , hasta que  = 0  V es el valor buscado. Vm

(120 +

0,20 0,15 0,14

190,288 244,95 263,44

2,811 5 V2

) (V – 0,046 23) - 24,470 3 

0,153 77 0,103 77 0,093 77

24,470 3 24,470 3 24,470 3

4,79 0,95 0,23 Interpolando

V = 0,137 L/molg 6.c) El % de Error será: (0,144  0,137) x 100 0,144

%E = -

%E = - 5,07%

%E =  5,07

Algunas Ecuaciones Utilizadas  Densidad molar:  m 

mRT   RT  . Masa molar  M   M  ….  VP  M P 

(21,4)  Para calcular las propiedades del gas desalojado de un recipiente de volumen constante: V(P1 – P2) = (m1 – m2)  Van de Waals: ( P 

RT …….(21,6) M

a ) (V - b) = RT V2

AMADO DE DIOS ([email protected])[Escriba texto] Página 74

Autor: Amado Crisógono Castro Chonta  Clausius: P =

a RT 2 V  b T (V  c )

 Lorentez: P =

RT a (V +b) - 2 2 V V

 Berthelot: P =

a RT V  b TV 2

 Keyes: P =

A RT ; (V  1) 2 V S

donde:

2015-2

S = e - /V

 Ecuación virial de estado.- Es una ecuación de la forma: B (T ) C (T ) PV =1+ + + ……. RT V V2

AMADO DE DIOS ([email protected])[Escriba texto] Página 75

Autor: Amado Crisógono Castro Chonta

2015-2

 BIBLIOGRAFÍA 1. J. Smith y N. Van Ness. “Introduction to Chemical Engineering Theromodynacies”. Editorial Mc Graw Hill. 7º Edición; 2003, México. 2. M. C. Reid y T. K. Sterwood. “The Proporties of Gases and Liquids. Editorial Mc Graw Hill. 3º Edición; New York 1970. 3. D. Mimelblan. “Principios y Cálculos Básicos de la Ingeniería Química”. Editorial CMCSA. 9º Edición; México 2003. 4. R. M. Perry y C. M. Chilton. “Chemical Engineer's Mandbook. Editorial Mc GRaw Hill – USA. 5º Edición; USA 2001 5. L. C. Nelson y E. I. Obert. “Chemical Engeneering”. Vol 61; 7. Pag. 203 a 208 (1989) 6. G. J. Van Wylen y R. E. Sountag. “Fundamentos de Termodinámica”. Editorial Limusa Wiley S.A. 7º Edición, México 2001. 7. Massermann y Otros. “propiedades termofísicas del Aire y de sus componentes”. Editorial Nauta, 1966. 8. J. Aguilar Peris. “Termodinámica”. Editorial Alambra S.A. España - Madrid; 1981.

AMADO DE DIOS ([email protected])[Escriba texto] Página 76

Autor: Amado Crisógono Castro Chonta

2015-2

.

AMADO DE DIOS ([email protected])[Escriba texto] Página 77

Autor: Amado Crisógono Castro Chonta

2015-2

AMADO DE DIOS ([email protected])[Escriba texto] Página 78

Autor: Amado Crisógono Castro Chonta

2015-2

AMADO DE DIOS ([email protected])[Escriba texto] Página 79

Autor: Amado Crisógono Castro Chonta

2015-2

AMADO DE DIOS ([email protected])[Escriba texto] Página 80

Autor: Amado Crisógono Castro Chonta

2015-2

TABLA DE FACTORES DE CONVERSIÓN MASA (M): unidad de medida en el SI (Kg) 1,0 UTM 1,0 slug 1,0 Kg 1,0 lb 1,0 ton 1,0 TM

= = = = = = =

9,81 32,2 1000 16,0 2000 1000 2205

LONGITUD (L): 1,0 km 1,0 m

= = = = 1,0 cm = 1,0 pie = = 1,0 plg = 1,0 yarda = 1,0 milla terrestre 1,0 milla marina 1,0 Aº = 1,0 u =

1,0 m3 1,0 pie3 1,0 bbl 1,0 gal us

= = = = = = =

DENSIDAD (D) : 1.0 g/cm3

= = =

FUERZA (F) : 1,0 kgf 1,0 N 1,0 lbf

m cm Aº plg Aº plg mm cm pie 1609 1852 cm cm

= = = = = =

g g kg kg slug

= m m = =

103 102 106 3,2808 104 0,3048

= =

2,205 lb 4,536 x 106 mg

= =

106 g = 109 mg 101,95 UTM

103 106 264,172 0,028 317 1 728 42 0,133 7

m cm  pie  m

= = = = = =

105 cm = 106 mm 103 mm 109 m 1,0936 yardas 107 m 30,48 cm

25,4 mm 91,44 cm = 5 280 pie = 6 076,3 pie 10-10 m 104 Aº

unidad de medida dm3 cm3 gal us m3 plg3 gal us pie3

SI = = = = = =

(m3) 103 L 106 mL

7,481 28,3168 5,615 3,785

= =

gal us L pie3 = L

1000 L 35,3147 pie3

159,0 L

unidad de medida en el SI (Kg/m3) 1000 kg/m3 62,427 8 lb/pie3 8,345 4 lb/gal

= =

103 kg/m3 3,612 7 x 10-2 lb/plg3

= 103g/L

unidad de medida en el SI (N)

= 9,81 N = 9,81 x 105 dyn = 105 dyn = 4,448 2 N

PRESIÓN (P) :

103 453,6 907,03 103 68,5

= = = = =

unidad de medida en el SI (m)

1000 100 1010 39,37 108 12 304,8 2,54 3,0 = = 10-8 10-4

VOLUMEN (V):

kg lb g oz lb kg lb

= = = =

9,81 Kg.m/s2 103 gf 105 g.cm/s2 4,482 x 105 dyn = 32,174 lb.pie/s2

unidad de medida en el SI (Pa)

AMADO DE DIOS ([email protected])[Escriba texto] Página 81

Autor: Amado Crisógono Castro Chonta 1,0 atm

1,0 bar 1,0 kgf/cm2 1,0 mm H2O 1,0 mmHg 1,0 lbf/pie2 1,0 lbf/plg2

= = = = = = = = = = = = =

33,91 10,333 76,0 1,033 3 14.696 1,013 25 101 325 105 750,061 735 9,81 133,3 47,88 6 895

2015-2

pie H2O = 406,92 plg H2O m H2O = 29,921 plg Hg cm Hg = 760 mm Hg kgf/cm2 = 103 33 Kgf /m2 Lbf/plg2 abs = 14.696 psia bar = 101 325 N/m2 Pa = 1 013 250 dyn/cm2 2 5 N/m =10 Pa = 106 dym/cm2 mmHg = 14,503 8 psia mmHg = 9,81 x 104 Pa = 10 m H2O Pa = Pa = Pa = 47,88 x 10-5 bar Pa = 0,068 95 bar

ENERGÍA: unidad de medida en el SI (J) 1,0 J

1,0 cal

1,0 kcal 1,0 Btu 1,0 L.atm 1,0 pie.lbf 1,0 m. kgf 1,0 ev

= = = = = = = = = = = = = = = = =

1,0 mN = 1,0 W.s = 1,0 kg.m2/s2 107 ergio = 107 dyn. cm 3 10 cm bar = 102 L.bar = 10-5m3 bar 3 9,869 23 cm .atm = 9,869 23 x 10-5 m3 bar -3 3 5,121 97 x 10 psia.pie = 0,239 006 cal 0,737562 pie.lbf = 9,478 31 x 10-4 Btu 4,184 J = 4,184 x 107 erg 3 41.84 cm bar = 0.041 84 L.bar 41,293 cm3.atm = 0,041 293 L.atm 0,021 43 psia.pie3 = 3,086 1 pie.lbf 103 cal = 4 184 J 1055 J = 778,16 pie.lbf = 252,15 cal 10,412 L.atm = 107,587 m.Kgf 10,333 m kgf = 24,214 cal 12,0 Plg.lbf = 1,355 82 J = 0,324 05 cal 9,806 J 1,602 x 10-12 erg = 10-6 Mev

POTENCIA: unidad de medida en el SI (W) 1,0 kw

1,0 HP 1,0 CV 1.0 Btu/s

= = = = = = = = = =

103W = 103 J/s = 103 VA = 103 kg.m/s3 3,6 x 106 J/h = 239,006 cal/s 737,562 pie.lbf.s-1 = 860,4 kcal/s 56,869 9 Btu,min-1 = 1,341 02 HP 3412,2 Btu.hr-1 2544,5 Btu.hr-1 = 745,7 W = 745,7 J/s 550 pie.lbf/s = 33 000 pie.lbf/min 760,0 m.kgf/s 735,7 w = 735,7 J/s = 633 kcal/h 1,055 kw

CONSTANTE UNIVERSAL DE LOS GASES IDEALES (R) R

= = =

0,082057 l.atm/molg K = 82,057 cm3.atm/molg K 8,317 J/molg ºK = 8,317 x 107 erg/molg K = 8,314 4 m3.Pa/molg K 1,987 cal/molg ºK = 1,987 Btu/mol lb R

AMADO DE DIOS ([email protected])[Escriba texto] Página 82

Autor: Amado Crisógono Castro Chonta = = = = = = =

2015-2

21,9 (plg Hg) pie3/mol lb R = 0,083 144 L.bar/molg K 62,361 L.mmHg/molg K = 0,730 2 pie3.psia/mol lb R 10,731 pie3.psia/mol lb R = 0,0848 (l) (kgf cm-2)/molg K 998,9 pie3.mmHg/mol lb K = 1,314 pie3.atm/mol lb K 555 pie3.atm/mol lb R = 1 545 pie.lbf/mol lb R 7,805 x 10-4 hp.hr/mol lb R = 5,819 x 10-4 kw.hr/mol lb ºR 1,851 x 104 plg lbf/mol lb ºR= =

44,568 (pie H2O) (pie3)/mol lb ºK

CONSTANTES: Gravedad (g) y

gc

9,8066 m/s2

98 066 kg.m/kgf.s2

980,66 cm/s2

32,174 lb.pie/lbf.s2

32,174 pie/s2

1,0 kg.m/Ns2 1,0 g.cm/dym.s2

CAPACIDAD CALORÍFICA : unidades en el SI (J/mol.K) Energía especifica: unidades en el SI (J/Kg) 1,0 cal/g K = 1.0 Btu/lb ºF = 4,184 J/g ºC Capacidad calorífica molar 1,0 cal/molg K = 1,0 Btu/mol lb ºR = 4,184 J/molg ºK Calor y entalpia especifica: 1,0 Btu/lb = 0,5559 cal/g = 2 325,89 J/kg 1,0 cal/g = 4,184 J/g = 4 184 J/kg = 1,8 Btu/lb

Viscosidad absoluta: unidades en el SI (Pa.s) 1,0 Pa.s

= 1,0 N.s/m2 = 10 dym.s/cm2 = 10 p = 0,672 1 lb/pie.s = 0,102 kgf.s/m2

1.0 p

= 1,0 g/cm.s = 0,1 kg/m.s

1,0 cp = 0,01 p = 2.4195 lb/pie.h VISCOSIDAD CINEMÁTICA: unidades en el SI (m2/s) 1,0 st

= 1,0 cm2/s = 10-4 m2/s

1,0 m2/s

= 10,763 9 pie2/s

CONDUCTIVIDAD TÉRMICA: unidades SI (w/ms) 1,0 Btu/pie.h.ºF

= 1,729 7 W/mk

AMADO DE DIOS ([email protected])[Escriba texto] Página 83

Autor: Amado Crisógono Castro Chonta

2015-2

= 1,488 3 kcal/m.h. ºC 1,0 kcal/m.h. ºC

= 1,162 2 W/mk

COEFICIENTE DE PELICULA O COEFICIENTE DE TRANSFERENCIA DE CALOR w/m2k 1,0 kcal/m2 hº C

= 1,162 2 W/m2k

1,0 Btu/pie2 hº F

= 5,675 W/m2k = 4.8830 kcal/m2 h ºC

UTM  unidad técnica de masa en el sistema Inglés Slug  unidad técnica de masa en el sistema británico Ton  tonelada corta. TM  tonelada métrica. plg  pulgada bbl  barril. gal us  galón americano. lb  libra masa lbf  libra peso. dym  DINA. Pa  pasacal. J  youl. W  watt. U  micro HP  caballo de fuerza. CV  caballo de vapor

AMADO DE DIOS ([email protected])[Escriba texto] Página 84

Autor: Amado Crisógono Castro Chonta

2015-2

PROBLEMAS 1. Calcular las presiones dadas por las ecuaciones de un gas ideal y de Van Der Waals para 10 mol de CO2 que ocupa un volumen de 3,81 x 10 -3 m3 a 40ºC. Si el vapor experimental es de 50 atm; calcular el % de error para cada caso? 2. Calcular los volúmenes por las ecuaciones de estado de los gases ideales, Van Der Waals y del factor de compresibilidad para 5 mol de N2 a 400 atm y 0ºC. Comparar y calcular los % de error, si el valor experimental es 3,515 x 10-4 m3. 3. La ecuación de Berthelot es (P + a/Tv2) (v-b) = RT. Demostrar: i) a = (16/3) PcTcV2c iii) Pv = RT (1 +

;

ii) b = (1/4) Vc

9 PTc (1-6T2c/T2)) 128 Pc T

4. Demostrar que a presiones moderadas y bajas la ecuación de VDW se puede escribir en la forma Pv = RT (1-BP); donde B = (1/RT) (a/RT-b). sugerencias: despreciar el término ab/v2 que es pequeño si la presión no es grande y sustituir v = RT/P en a/v. 5. Un cilindro de 100,0 x 10-3 m3 de capacidad contiene gas metano a 200,0 atm y 25ºC Determinar el peso de gas gastado cuando la presión en el cilindro ha caído a 50,0 atm, por tres métodos: a) gas ideal, b) VDW, y c) factor Z. 6. Usando los datos de temperatura y presión crítica del etano calcular las constantes de VDW y de Berthelot. Cuál sería el volumen molar crítico y comparar con el vapor experimental? 7. Calcular los volúmenes por: i) gas ideal; ii) VDW y iii) factor de compresibilidad para 5.5 mol de H2 a 500 atm y 250 K. 8. Una mezcla formada por 46,0 g de O 2 y 154,0 g de N2 ocupan 310 x 10-3 m3 a 273.15 ºK. Calcular la presión por los métodos de los: i) gases ideales, ii) VDW + Dalton; iii) VDW + Amaget; iiii) VDW y las constantes medias; iiiii) Factor Z + Amagat; 6i) Factor Z + Dalton y 7i) Kay. 9. Hallar la forma reducida de la ecuación de estado de Berthelot: (P+a/Tv2) (v-b) ) RT. 10. Determinar las constantes críticas de un gas que obedece la siguiente ecuación de estado: P = RT/(v-b)-a/T(v-c)2. 11. Un gas real posee las siguientes densidades a 300,0 K; Presión Densidad (g/l)

P(atm) 0,40 1,512

0,80

1,00

3,088

3,900

AMADO DE DIOS ([email protected])[Escriba texto] Página 85

Autor: Amado Crisógono Castro Chonta

2015-2

Calcular el peso molecular del gas? 12. Usando los parámetros críticos Pc y Tc determinar los valores de las constantes a y b de las ecuaciones de estado de berthelot y VDW de los siguientes gases: a) agua; b) SO2; c) etanol d) amoniaco; e) CS2; f) CCl4 y g) C6H6. 13. Hallar los valores de las constantes a , b y c de la ecuación empírica de Wohl; usando los parámetros críticos Pc y Tc de los gases propuestos en el problema 12. 14. Determinar los valores de las constantes a y b de la ecuación de estado de Dieterici y representar ésta en forma reducida. 15. Calcular la densidad molar de la siguiente mezca gaseosa a 600,3 psig y 300ºF: metano 100 lb; etano 240 lb; propano 150 lb; y N2 50 lb; mediante los métodos propuestos en el problema 8. 16. El análisis de un gas indica 50,5% de metano y el resto etileno a 1 atm. Se desea almacenar 25 kg de esta mezcla gaseosa en un cilindro cuya capacidad es de 3,26 pie3 a una temperatura máxima de 320 K. Calcular la presión dentro del cilindro por los siguientes métodos: a) gas ideal; b) VDW y las constantes medias; c) factor Z + Amagat; d) Kay. Cuántas libras de esta mezcla pueden almacenarse en el cilindro a 320 K si la máxima presión permisible es de 1350 psig? 17. Se va llevar gas natural por una tubería de 30,54cm de diámetro interno. El metano puro entra al gaseoducto a razón de 70 lb/s a una presión de 3,000 lb f/plg2 y a una temperatura de 70ºF. Calcular la densidad incivil en el Si suponiendo que el metano obedece a la ecuación de estado de: a) gas ideal; b) VDW; y c) factor Z.

AMADO DE DIOS ([email protected])[Escriba texto] Página 86

Autor: Amado Crisógono Castro Chonta

2015-2

AMADO DE DIOS ([email protected])[Escriba texto] Página 87